Tax Chapter 5

Pataasin ang iyong marka sa homework at exams ngayon gamit ang Quizwiz!

Geoff purchased a life annuity for $4,800 that will provide him $100 monthly payments for as long as he lives. Based on IRS tables, Geoff's life expectancy is 240 months. How much of the first $100 payment will George include in his gross income? $ 100 $ 80 $ 48 $ 20 None of the choices are correct.

$ 80

Geoff purchased a life annuity for $4,800 that will provide him monthly payments of $100 as long as he lives. Based on IRS tables, Geoff's life expectancy is 240 months. How much of the first $100 payment will George include in his gross income? $ 100 $ 80 $ 48 Incorrect $ 20 None of the choices are correct.

$ 80 ?

Chester incurred $14,500 in hospital and medical bills during the current year. His health insurance policy reimbursed him $11,600 toward those expenses. What amount should be included in Chester's gross income?

$0

Chester incurred $14500 in hospital and medical bills during the current year. His health insurance policy reimbursed hum $11600 toward those expenses. What amount should be included in Chester's gross income?

$0

Clyde is a cash-method taxpayer who reports on a calendar-year basis. This year Paylate Corporation has decided to pay Clyde a year-end bonus of $1,000. Determine the amount Clyde should include in his gross income this year under the following circumstances Clyde picked up the check in December, but the check could not be cashed immediately because it was postdated January 10.

$0

Grady received $8,200 of Social Security benefits this year. Grady also reported salary and interest income this year. What amount of the benefits must Grady include in his gross income under the following five independent situations? Grady files single and reports salary of $12,100 and interest income of $250.

$0

Jamarcus injured his hand playing softball one weekend. The injury prevented him from being able to work. Luckily for Jamarcus, he had purchased a disability policy for himself easier in the year. Jamarcus received $1300 in disability benefits while he was away from work. Jamarcus should include _____ in gross income.

$0

Ralph owns a building that he is trying to lease. Ralph is a calendar-year, cash-method taxpayer and is trying to evaluate the tax consequences of three different lease arrangements. Under lease 1, the building rents for $500 per month, payable on the first of the next month, and the tenant must make a $500 security deposit that is refunded at the end of the lease. Under lease 2, the building rents for $5,500 per year, payable at the time the lease is signed, but no security deposit is required. Under lease 3, the building rents for $500 per month, payable at the beginning of each month, and the tenant must pay a security deposit of $1,000 that is to be applied toward the rent for the last two months of the lease. What amounts are included in Ralph's gross income this year if a tenant signs lease 1 on December 1 and makes timely payments under that lease?

$0

Todd and Margo are seeking a divorce and no longer live together. Margo has offered to pay Todd $42,000 per year for five years if Margo receives sole title to the art collection. This collection cost them $100,000 but is now worth $360,000. All other property is to be divided equally. How much of the gain would be taxed to Todd if Margo sells the art at the end of five years?

$0 - Margo is the sole owner

Louis files as a single taxpayer. In April of this year he received a $900 refund of state income taxes that he paid last year. How much of the refund, if any, must Louis include in gross income under the following independent scenarios? Assume the standard deduction last year was $6,300. Last year Louis had itemized deductions of $4,800 and he chose to claim the standard deduction. Louis's itemized deductions included state income taxes paid of $1,750.

$0 - his SD was greater than itemized

Clyde is a cash-method taxpayer who reports on a calendar-year basis. This year Paylate Corporation has decided to pay Clyde a year-end bonus of $1,000. Determine the amount Clyde should include in his gross income this year under the following circumstances Paylate Corporation mailed the check to Clyde before the end of the year (and it was delivered before year-end). Although Clyde expected the bonus payment, he decided not to collect his mail until after year-end.

$1,000

Clyde is a cash-method taxpayer who reports on a calendar-year basis. This year Paylate Corporation has decided to pay Clyde a year-end bonus of $1,000. Determine the amount Clyde should include in his gross income this year under the following circumstances Paylate Corporation wrote the check and put it in his office mail slot on December 30 of this year, but Clyde did not bother to stop by the office to pick it up until after year-end.

$1,000

Mary has received the following income and fringe benefits during the current year: a $73,000 salary; $600 in employer-provided disability premiums; $5,000 in workers' compensation; $1,200 in corporate bond interest; a $23,000 car won on a game show; and a $1,200 health insurance reimbursement for medical expenses paid during the year. Which of these amounts should Mary include in her gross income?

$1,200 corporate bond interest $73,000 salary

Jamarcus injured his hand playing softball one weekend. The injury prevented him from being able to work. Luckily for Jamarcus, his employer provides disability insurance as a nontaxable fringe benefit to all employees. Jamarcus received $1,300 in disability benefits while he was away from work. Jamarcus should include $_________ in gross income.

$1,300 Since the employer paid the disability premiums, the benefits received are taxable

Ralph owns a building that he is trying to lease. Ralph is a calendar-year, cash-method taxpayer and is trying to evaluate the tax consequences of three different lease arrangements. Under lease 1, the building rents for $500 per month, payable on the first of the next month, and the tenant must make a $500 security deposit that is refunded at the end of the lease. Under lease 2, the building rents for $5,500 per year, payable at the time the lease is signed, but no security deposit is required. Under lease 3, the building rents for $500 per month, payable at the beginning of each month, and the tenant must pay a security deposit of $1,000 that is to be applied toward the rent for the last two months of the lease. What amounts are included in Ralph's gross income this year if the tenant signs lease 3 on December 31 and makes timely payments under that lease?

$1,500

Ben's employer offers employees the following benefits. What amount must Ben include in his gross income? Health insurance coverage $ 5,800 Group term life insurance (face $50,000) 4,270 Disability insurance coverage (considered purchased by Ben) 3,600 Whole life insurance coverage ($100,000) 7,000

$10,600

Clyde is a cash-method taxpayer who reports on a calendar-year basis. This year Paylate Corporation has decided to pay Clyde a year-end bonus of $1,000. Determine the amount Clyde should include in his gross income this year under the following circumstances Paylate Corporation mistakenly wrote the check for $100. Clyde received the remaining $900 after year-end.

$100

Bonnie and Howard are getting divorced. Under the terms of the decree Bonnie will pay Howard $100,000 in cash in each of the next ten years (or until Howard's death or remarriage). In addition, Bonnie will transfer a residence worth $2,000,000 to Howard and pay $30,000 per year to support their daughter, Kristina, until she turns 19 years old. What amount (if any) is included in Howard's gross income this year? $2,130,000 $100,000 $500,000 $130,000 None of the payments

$100,000

Bonnie and Howard got divorced in 2018. Under the terms of the decree Bonnie will pay Howard $100,000 in cash in each of the next ten years (or until Howard's death or remarriage). In addition, Bonnie will transfer a residence worth $2,000,000 to Howard and pay $30,000 per year to support their daughter, Kristina, until she turns 19 years old. What amount (if any) is included in Howard's gross income this year?

$100,000 Property settlements and child support are not included in gross income

exclusion amount is limited to _______ for foreign earned income

$102,100

Helen is a U.S. citizen and CPA, who moved to London, England three years ago to work for a British company. This year, she spent the entire year in London and earned a salary of $110,000. How much of her salary will she be allowed to exclude from gross income in the U.S.? $82,000. $103,900. $105,500. $108,000. All of her salary is included in gross income.

$103,900

a maximum of _______ in 2017 of employer-provided foreign housing may also be excluded

$14,294

NeNe is an accountant and U.S. citizen, who has accepted a permanent position in Madrid, Spain for a Spanish financial services company. This year, NeNe spent the entire year working in Madrid. NeNe's employer paid $40,000 of her Madrid housing expenses this year. What amount of the $40,000 housing payments may NeNe exclude? NeNe can exclude all of the housing payment because she worked more than 330 days overseas. - INCORRECT 16,624 23,376 14,546 None of her salary can be excluded from gross income.

$14,546 Since NeNe spent the entire year overseas, she may exclude the lesser of (a) $40,000 housing costs − $16,208 ($101,300 × .16) = $23,792 or (b) $14,182 ($101,300 × .14). Thus, she may exclude $14,182 of the housing costs paid by her employer.

Rhett made his annual gambling trip to Uwin Casino. On this trip Rhett won $250 at the slots and $1,200 at poker. Also this year, Rhett made several trips to the racetrack, but he lost $700 on his various wagers. What amount must Rhett include in his gross income?

$1450 - Winnings go into gross income, not the losses on wagers

Deb has found it very difficult to repay her loans. Because of these difficulties, the bank decided to forgive one of her most recent loans, an amount of $45,000. After the loan was discharged, Deb had total assets of $232,000 and her remaining loans total $217,000. What amount must Deb include in her gross income?

$15,000 If debt relief makes you solvent, then you recognize the amount your assets is greater than liabilities If still insolvent, recognize amount of debt forgiveness

This year Zach was injured in an auto accident. As a result, he received the following payments. Zach received $18,000 of disability pay. Zach has disability insurance provided by his employer as a nontaxable fringe benefit. Zach's employer paid $4,300 in disability premiums for Zach this year. Zach's hospital bills totaled $4,500 and were paid by his health insurance. Zach has health insurance provided by his employer as a nontaxable fringe benefit. Zach's employer paid $6,250 in health insurance premiums for Zach this year. What amount must Zach include in his gross income?

$18,000 Any payment a taxpayer receives from a health and accident insurance policy for medical or dental expenses paid by the taxpayer is excluded from the taxpayer's income. If the employer pays the disability premiums for an employee as a nontaxable fringe benefit, the employee must include all disability benefits in gross income. Zach's employer pays both health insurance and disability insurance premiums... so the entire amount of disability insurance that he receives must be included in Gross income.

Bart sold a parcel of land for $21000. He paid a real estate agent a commission of $1500 for assisting with the sale. Bart had purchased the land several years earlier for $20000, What is the amount realized on the sale of the land?

$19500

Although Hank is retired, he is an excellent handyman and often works part-time on small projects for neighbors and friends. Last week his neighbor, Mike, offered to pay Hank $500 for minor repairs to his house. Hank completed the repairs in December of this year. Hank uses the cash method of accounting and is a calendar-year taxpayer. Mike paid Hank $200 in cash in December of this year and promised to pay the remaining $300 with interest in three months.

$200

This year Mary received a $200 refund of state income taxes that she deducted on her tax return last year. Mary included a total of $4,000 of state income taxes when she itemized deductions last year. What amount of the refund, if any, should Mary include in her gross income this year?

$200 is included if itemized deductions exceeded the standard deduction by $200. -refund amounts are included in gross income only to the extent that the original deduction provided a tax benefit. The $4,000 of deductions produced a tax benefit of $200 it itemized deductions exceeded the standard deduction by $200

Ethan competed in the annual Austin Marathon this year and won a $25,000 prize for fastest wheelchair entrant. Ethan indicated that he would transfer the prize to the local hospital. How much of the prize should Ethan include in his gross income?

$25,000 Prizes, awards and gambling winnings are included in gross income. Two exceptions: 1. Awards for scientific, literary, or charitable achievement such as the Nobel Prize are excluded from gross income. But only if 1. The recipient was selected without any action on his part to enter the contest (Ethan entered himself in the marathon). 2. The recipient is not required to render substantial future services as a condition to receive the prize or award. 3. The payor of the prize transfers the the prize or award to a federal, state, or local government unit or a qualified charity.

Single taxpayers meeting certain home ownership and use requirements can permanently exclude up to _____ of the realized gain on the sale of their principal residence.

$250,000

Single taxpayers meeting certain home ownership and use requirements can permanently exclude up to __________ of the realized gain on the sale of their principal residence.

$250,000

Graham has accepted an offer to do graduate work in the chemistry department at State University. The chemistry department offered Graham a $5,000 tuition reduction and $3,500 toward the cost of room and meals. Under the terms of the scholarship Graham must work in the chemistry labs during the summer as a research assistant. What amount must Graham include in his gross income?

$3,500 -because the tuition reduction is covered as a nontaxable option according to the code but the money towards room and board are considered excess scholarship amounts

Frick and Frack are equal partners in F&F Industries. The company generated net income of $60000 during the year and distributed $7000 cash to both Frick and Frack. How much income should Frack report on his individual income tax return due to the partnership interest?

$30,000

Frick and Frack are equal partners in F&F Industries. The company generated net income of $60,000 during the year and distributed $7,000 cash to both Frick and Frack. How much income should Frack report on his individual income tax return due to the partnership interest?

$30,000 Distributions to partners are not taxable. Only the portion of the net income allocable to the partner is taxable.

During the current year, Sam received interest income from the following investments: $400 from State of Wyoming bonds, $200 from Ford Motor Co, $50 from City of Laramie bonds, $100 from US Treasury bonds. How much of the interest received will be included in gross income/

$300

Sam traded a parcel of land for a tractor and a car. He had purchased the land five years earlier for $16,000. The market value of the car and the tractor is $20,000. What is the amount of gross income resulting from this transaction?

$4,000

Although Hank is retired, he is an excellent handyman and often works part-time on small projects for neighbors and friends. Last week his neighbor, Mike, offered to pay Hank $500 for minor repairs to his house. Hank completed the repairs in December of this year. Hank uses the cash method of accounting and is a calendar-year taxpayer. Compute Hank's gross income for this year from each of the following alternative transactions: Mike gave Hank tickets in December to the big game in January. The tickets have a face value of $50 but Hank could sell them for $400. Hank went to the game with his son.

$400

Pam recently was sickened by eating spoiled peanut butter. She successfully sued the manufacturer for her medical bills ($3,700), her emotional distress ($6,000 - she now fears peanut butter), and punitive damages ($44,000). What amount must Pam include in her gross income?

$44,000 The tax laws specify that any payments (OTHER THAN PUNITIVE DAMAGES) on account of a physical injury or physical sickness are nontaxable. Damages taxpayers receive for emotional distress associated with a physical injury are also excludable. Punitive damages are fully taxable, however, because they are intended to punish the harm-doer rather than to compensate the taxpayer for injuries.

Although Hank is retired, he is an excellent handyman and often works part-time on small projects for neighbors and friends. Last week his neighbor, Mike, offered to pay Hank $500 for minor repairs to his house. Hank completed the repairs in December of this year. Hank uses the cash method of accounting and is a calendar-year taxpayer. Compute Hank's gross income for this year from each of the following alternative transactions: Mike bought Hank a new set of snow tires. The tires typically sell for $500, but Mike bought them on sale for $450.

$450

George purchased a life annuity for $3,200 that will provide him $80 monthly payments for as long as he lives. Based on IRS tables, George's life expectancy is 100 months. How much of the first $80 payment will George include in his gross income?

$48 The annuity exclusion ratio is ($3200/100) =32 return of capital per payment. Hence, $48 of the $80 is included in gross income

Ralph owns a building that he is trying to lease. Ralph is a calendar-year, cash-method taxpayer and is trying to evaluate the tax consequences of three different lease arrangements. Under lease 1, the building rents for $500 per month, payable on the first of the next month, and the tenant must make a $500 security deposit that is refunded at the end of the lease. Under lease 2, the building rents for $5,500 per year, payable at the time the lease is signed, but no security deposit is required. Under lease 3, the building rents for $500 per month, payable at the beginning of each month, and the tenant must pay a security deposit of $1,000 that is to be applied toward the rent for the last two months of the lease. What amounts are included in Ralph's gross income this year if the tenant signs lease 2 on December 31 and makes timely payments under that lease?

$5,500

group term life insurance exclusion amount for employer payments

$50,000

Although Hank is retired, he is an excellent handyman and often works part-time on small projects for neighbors and friends. Last week his neighbor, Mike, offered to pay Hank $500 for minor repairs to his house. Hank completed the repairs in December of this year. Hank uses the cash method of accounting and is a calendar-year taxpayer. Mike paid Hank $100 in cash in December of this year and gave him a negotiable promissory note for $400 due in three months with interest. Hank sold the note in January for $350.

$500

This year, Barney and Betty sold their home (sales price $750,000; cost $200,000). All closing costs were paid by the buyer. Barney and Betty owned and lived in their home for 18 months. Assuming no unusual or hardship circumstances apply, how much of the gain is included in gross income?

$550,000 In this case the Property is held by the owner for less than 36 months. Thus the gain on the property sold will be considered under Short term Capital Gains. Full Consideration = $750,000 Less: Cost of Aquisition = $200,000 Gross Short term Capital Gain = $550,000

Daniels, Edwards, and Findley are equal partners in DEF industries. The company generated net income of $180,000 during the year and distributed $20,000 cash each of the partners. Edwards will report $__________ in taxable income from the partnership.

$60,000

Hal Gore won a $1 million prize for special contributions to environmental research. This prize is awarded for public achievement, and Hal directed the awarding organization to transfer $400,000 of the award to the Environmental Protection Agency. How much of the prize should Hal include in his gross income?

$600,000

Andrea owed $12000 on a medical bill to University Hospital. The hospital agreed to discharge the debt due to Andrea's financial situation. Immediately prior to the discharge of the debt, Andrea's debts exceeded her assets by $5000. How much of the debt forgiveness will Andrea need to include in her gross income?

$7000

Bart, a single taxpayer, has recently retired. This year, he received $24,000 in pension payments and $5,000 of social security payments. What amount must Bart include in his gross income for the social security payments?

$750 If MAGI +50% of SSB <= 25,000 SSB are not taxable. Our example does not pass this 50%(24000+2500-25000)= 750

Jack and Jill are married. This year Jack earned $72,000 and Jill earned $80,000 and they received $4,000 of interest income from a joint savings account. How much gross income would Jack report if he files married-filing-separate from Jill?

$78,000 if they reside in a community property law state.

Brenda received $15000 college scholarship for the current year. She used the scholarship to pay the following fees: tuition $9000, lab fees $60, parking fees $120, books $750, housing $4000, and a meal plan $1070. What can can she exclude from gross income?

$9000 tuition, $60 lab fees, $750 books

Alimony Definition

(1) a transfer of cash made under a written separation agreement or divorce decree, (2) the separation or divorce decree does not designate the payment as something other than alimony, (3) in the case of legally separated (or divorced) taxpayers under a separation or divorce decree, the spouses do not live together when the payment is made, and (4) the payments cannot continue after the death of the recipient.

Types of annuities

(1) annuities paid over a fixed period and (2) annuities paid over a person's life.

Which of the following represent economic benefits to a taxpayer?

- a computer received in exchange for services rendered - interest income on investments - cash received for completing a job

Under the realization principle, what must occur for income to be realized?

- a taxpayer must engage in a transaction with another party - the transaction must result in a measurable change in property rights

Which of the following fringe benefits are excluded from taxation in 2018?

- group term life insurance under $50,000 - dependent care benefits - medical insurance

Under which of the following circumstances might life insurance proceeds be included in gross income?

- when a life insurance policy is cashed out by the insured before death and the proceeds exceed the premiums paid - when a life insurance policy is transferred to another party for valuable consideration

income from partnerships

-a partnership is not a separate taxable entity -files an information return (form 1065)

exemption amount for tax avoidance loan

0

what % of tax do individuals pay who are in the 10% or 15% tax bracket

0%

What are the itemized deductions? (3 categories with 3 each)

1) Business Deductions a) Trade or Business Deductions b) Depreciation c) Qualified Trade or Business 2) Non-Business Deductions a) Interest b) Taxes c) Losses 3) Personal Deductions a) Expenses for the Production of Income (Investment Expenses) b) Alimony (Before 2019) c) Alimony (After 2018)

What are the accounting methods?

1) Cash 2) Accrual

What are the 5 basic steps to analyzing a tax question?

1) Determine Gross Income 2) Deductions 3) Characterization 4) Apply rates (progressive) to taxable income to arrive at tax liability 5) Subtract credits from tax liability to reach net tax due

What are the steps for analyzing gains from property? (4)

1) Determine gain (or loss) realized 2) Determine if gain (or loss) is recognized 3) Determine if gain is excluded (for losses must determine if deductible) 4) Characterize gain (or loss)

Calculation for finding gross income of each annuity payment (life annuity) textbook

1) Investment in annuity contract 2)Expected return multiple (Exhibit 5-1, "70" years old) 3)Amount of each payment 4)Expected return (2)x(3) X 12 MONTHS 5)ROC Percentage (1)/(4) 6)ROC per payment (3)x(5) TAXABLE INCOME PER PAYMENT: (3)-(6)

What are the four filing status?

1) Joint return (To file a joint return, the parties must be married at the end of the year. If one spouse dies during the year, a joint return may still be filed for that year.) 2) Head of Household 3) Single 4) Married Filing Separately

What are the two types of dependents?

1) Qualifying child 2) Qualifying relative

exceptions to prizes and awards

1) taxpayer designates qualified organization to receive price or award 2) employee achievement awards of tangible personal property made in recognition of length of service or safety achievement - no cash - limit $400 per employee annually (excess is taxable)

If an individual suffers losses, those losses are deductible only if related to

1) trade or business 2) transactions entered into for profits or 3) due to fire, storm, shipwreck, other casualty or theft.

How is income defined?

1) undeniable accession to wealth 2) clearly realized and 3) over which the taxpayers have complete dominion

Problems with Alimony

1)Definition eliminates uncertainty regarding which transfers are treated as alimony 2)It is possible to use alimony transfers to assign income from the (higher tax rate) payer of the alimony to the (lower tax rate) recipient.

Social Security Benefits for Single taxpayers

1)If modified AGI + 50 percent of Social Security benefits ≤ $25,000, Social Security benefits are not taxable. 2)If $25,000 < modified AGI + 50 percent of Social Security benefits ≤ $34,000, taxable Social Security benefits are the LESSER of (a) 50 percent of the Social Security benefits or (b) 50 percent of (modified AGI + 50 percent of Social Security benefits − $25,000). 3)If modified AGI + 50 percent of Social Security benefits > $34,000, taxable Social Security benefits are the LESSER of (a) 85 percent of Social Security benefits or (b) 85 percent of (modified AGI + 50 percent of Social Security benefits − $34,000), PLUS LESSER of (1) $4,500 or (2) 50 percent of Social Security benefits.

Other Sources of gross income

1)Income from Flow-through Entities 2)Alimony 3)Prizes, Awards, and Gambling Winnings 4)Social Security benefits 5)Imputed Income 6)Discharge of Indebtedness

Calculation for finding gross income of each annuity payment (Fixed annuity) textbook

1)Investment in annuity contract 2)Number of payments 3)Return of capital per payment (1)/(2) 4)Amount received every year GROSS INCOME PER PAYMENT: (4)-(3)

Which of the following statements is INCORRECT regarding the receipt of Social Security benefits?

1. 50% of SS benefits are taxable to all taxpayers because the employer contributed funds that were never taxed to the employee 2. SS benefits are not taxable because the contributions were taxed when the taxpayer was working

When has the constructive receipt occurred

1. alex was out of town on dec 31 and unable to pick up his final paycheck. he picked it up Jan 2 2. arnold earned interest of $300 on his savings account, but did not withdraw the funds until the following year 3. amber received a year-end bonus check dated dec 23, but did not cash the check until jan 3

Which of the following types of income are generated from property ownership?

1. dividends received on corporate stock 2. rental income from lessees 3. interest earned on US Treasury bonds 4. gain from the sale of a building

what are the advantages of the realization principle for defining gross income?

1. it provides an objective measure of the value of the transaction 2. it provides the taxpayer with the wherewithal to pay when cash is received int he transaction

In order to exculde the maximum amount of foreign-earned income from US taxation, the following conditions must be met

1. the taxpayer must be considered a "resident" of the foreign country 2. the taxpayer must have resided in the foreign country for 330 days in consecutive 12-month period

What describes exclusions and deferrals for tax purposes?

1. these provisions are often granted in order to subsidize or encourage particular behaviors 2. these provisions are the result of specific congressional action 3. these provisions are narrowly defined

What is correct regarding the receipt of SS benefits?

1. up to 85% of SS benefits may be taxed to taxpayers with moderate to high taxable income 2. SS benefits are not taxable if the recipient has relatively low taxable income

Which of the following situations will result in an award being excluded from gross income?

1. when the award is a noncash item valued at less than $400, and given for either safety or years of service by an employee 2. when the award is given for scientific, literary, or charitable achievement and meets certain other requirements

Social Security Benefits for MFJ:

1.If modified AGI + 50 percent of Social Security benefits ≤ $32,000, Social Security benefits are not taxable. 2.If $32,000 < modified AGI + 50 percent of Social Security benefits ≤ $44,000, taxable Social Security benefits are the lesser of (a) 50 percent of the Social Security benefits or (b) 50 percent of (modified AGI + 50 percent of Social Security benefits − $32,000). 3.If modified AGI + 50 percent of Social Security benefits > $44,000, taxable Social Security benefits are the lesser of (a) 85 percent of Social Security benefits or (b) 85 percent of (modified AGI + 50 percent of Social Security benefits − $44,000), plus the lesser of (1) $6,000 or (2) 50 percent of Social Security benefits.

what is the penalty amount for early distributions for annuities

10% but penalty is waived for over 59.5 or disabled

Bonnie and Howard divorced in 2018. Under the terms of the decree Bonnie will pay Howard $100,000 in cash in each of the next ten years (or until Howard's death or remarriage). In addition, Bonnie will transfer a residence worth 2,000,000 to Howard and pay $30,000 per year to support their daughter, Krista, until she turns 19. What amount is included in gross income for Howard?

100000, Alimony Recipient

what % of tax do individuals pay who are in the 25, 28, 33, or 35% tax bracket

15%

george is a 50% partner in ABC. ABC reported $30,000 of business income and $5,000 of interest income this year. ABC also distributed $2,000 of cash to george. what amounts of gross income from his ownerships in ABC would george report for the current year?

15,000 + 2,500 = 17,500 (distributions are not taxed)

what % of tax do individuals pay who are in the 39.6% tax bracket

20%

maximum deduction for student loan interest

2500

Anne purchased an annuity from an insurance company that promised to pay her $20,000 per year for the next 10 years. Anne paid $145,000 for the annuity, and in exchange she will receive $200,000 over the term of the annuity. How much of the first $20,000 payment should Anne include in gross income?

5,500

Anne purchased an annuity from an insurance company that promised to pay her $20,000 per year for the next 10 years. Anne paid $145,000 for the annuity, and in exchange she will receive $200,000 over the term of the annuity. How much income will Anne recognize over the term of the annuity?

5,500 x 10 years = 55,000

The earnings on a Section _____ plan are not taxable to the beneficiary if the distributions from the account are used to pay qualified higher education expenses.

529

Louis files as a single taxpayer. In April of this year he received a $900 refund of state income taxes that he paid last year. How much of the refund, if any, must Louis include in gross income under the following independent scenarios? Assume the standard deduction last year was $6,300. Last year Louis claimed itemized deductions of $6,550. Louis's itemized deductions included state income taxes paid of $1,750.

6,550 - 6,300 = 250

Grady received $8,200 of Social Security benefits this year. Grady also reported salary and interest income this year. What amount of the benefits must Grady include in his gross income under the following five independent situations? Grady files married joint and reports salary of $75,000 and interest income of $500.

6,970

Louis files as a single taxpayer. In April of this year he received a $900 refund of state income taxes that he paid last year. How much of the refund, if any, must Louis include in gross income under the following independent scenarios? Assume the standard deduction last year was $6,300. Last year Louis claimed itemized deductions of $7,740. Louis's itemized deductions included state income taxes paid of $2,750.

7,740 - 6,300 = 1,440 BUT the answer is 900 can't exclude more than the refund amount

The highest percentage of SS benefits that may be taxed is ___ % and only for moderate to high income taxpayers

85

The highest percentage of social security benefits that may be taxed is __________ , and only for moderate to high income taxpayers.

85%

Grady received $8,200 of Social Security benefits this year. Grady also reported salary and interest income this year. What amount of the benefits must Grady include in his gross income under the following five independent situations? Grady files single and reports salary of $22,000 and interest income of $600.

850

Sam traded a parcel of land for a tractor and a car. He had purchased the land five years earlier for $16,000. The market value of the car and tractor is $20,000. What is the amount of gross income resulting from this transaction? a. $4,000 b. $20,000 c. $0 d. $16,000

A

Which of the following choices does NOT describe an annuity? a. It is a lump sum payment that is usually received by the beneficiary of a life insurance policy b. It is a means of generating a fixed income stream during retirement c. It is an investment that pays a stream of equal payments over time

A

Which of the following choices does not describe an annuity? a. It is a lump sum payment that is usually received by the beneficiary of a life insurance policy b. It is a means of generating a fixed income stream during retirement c. It is an investment that pays a stream of equal payments over time

A

Which one of the following choices states that income is realized if a taxpayer receives income and there are no restrictions on the taxpayer's use of the income? a. Claim of right doctrine b. Business purpose doctrine c. Income recognition doctrine d. Assignment if income doctrine

A

Which one of the following is NOT an advantage of the cash method for reporting income? a. Taxpayers are able to deduct expenses in the period incurred, which may be before they actually pay them b. Taxpayers recognize income in the period they receive it, giving them the wherewithal to pay the tax c. Taxpayers have some control over when income is received and expenses are paid which assets in tax planning d. The cash method generally simplifies the computation of income

A

Jerry receives an annuity payment of $2,500 per month. Jerry purchased the 20-yer annuity for $250,000. What is the amount of the annuity that represents a return of capital and is, therefore, nontaxable? a. 41.67% or $12,500 per year b. 100% or $30,000 per year c. 10% or $3,000 per year d. 12% or $3,600 per year

A $250,000/(20x2500x12)=0.4167

Bart sold a parcel of land for $21,000. He paid a real estate agent a commission of $1,500 for assisting with the sale. Bart had purchased the land several years earlier for $20,000. What is the amount realized on the sale of the land? a. $19,500 b. $21,000 c. $1,000 d. ($500)

A Amount realized= Sales proceeds - selling expenses

Which of the following types of imputed income are NOT included in gross income (i.e. are NOT taxable)? a. A bargain purchase between a father and his son b. An employer's $12,000 loan to an employee with no interest on the note c. Employee discounts of 25% on services

A A bargain purchase between family members is generally deemed to be a gift, thus nontaxable to the recipient

What is the rule for specified service trade or business?

A "specified service trade or business" is in the field of health, law, accounting, actuarial science, performing arts, consulting, athletics, financial services, brokerage services, or any trade or business where the principal asset of such trade or business is the reputation or skill of one or more of its employees.

What is an accounting method?

A consistent plan for reporting income.

Which of the following statements is correct concerning a gift?

A gift may be subject to "gift tax" which is paid by the person giving the gift.

Which of the following is a true statement about the first payment received from a purchased annuity? The payment is included in gross income. A portion of the payment is a return of capital. The payment can only be taxed in the year after the annuity was purchased. The payment is not taxed until the annuity payments cease altogether. None of these is a true statement.

A portion of the payment is a return of capital.

When analyzing gains from property, explain Step 3 determining if gain is excluded (for losses must determine if deductible).

A taxpayer may exclude the gain on the sale of exchange of property if 1) such property has been owned and used as the taxpayer's principal residence 2) for 2 out of the last 5 years 3) may use the exclusion only once every 2 years.

What is an assignment of income generally?

A taxpayer may not avoid income from services or property generally by merely assigning the right to the item of income to another taxpayer either directly or through an entity. Since the tax is progressive (the greater a taxpayer's taxable income, the higher the rate of taxation) there is an incentive to assign income to others in a lower tax bracket. Courts, not the Code, have dealt with this in the following manner.

Which of the following choices are characteristics of qualified tuition programs, also known as Section 529 plans? a. Earning on the account are not taxable if used for qualified higher education expenses b. The distributions can be made for qualified education expenses from kindergarten through 12th grade c. Distribution made to the beneficiary for purposed other than education will incur taxation and a penalty on the earnings of the plan d. The maximum yearly contributions to the account are limited to $2000 for each beneficiary e. Distributions to contributors are not subject to income tax, but they do incur a 10% penalty

A, B, C

Choose the types of investments that generate taxable income. a. Corporate bonds b. US Savings Bonds c. Educational savings plan d. Certificate of Deposits e. Municipal bonds

A, B, D

Under the realization principle, what must occur for income to be realized? a. A taxpayer must engage in a transaction with another party b. A taxpayer must receive cash to realize income c. The transaction must result in a measurable change in property rights d. A measurable change in wealth must occur, regardless of whether a transaction takes place

A, C

What are the advantages of the realization principle for defining gross income? a. It provides an objective measure of the value of the transaction b. It allows the taxpayer to exclude the transaction from gross income c. It provides the taxpayer with the opportunity to shift income to a related taxpayer who is in a lower tax bracket d. It provides the taxpayer with the wherewithal to pay when cash is received in the transaction

A, D

Cha has received the following income and benefits during the current year: $65000 salary, $4800 employer-provided health insurance, $1500 municipal bond interest, $2000 dividend income, $500 from a partnership, and a $10000 judgment for lost wages due to an age discrimination lawsuit. What amounts should Chad include in his gross income? a. $10000 judgment b. $4800 health insurance c. $1500 municipal bond interest d. $65000 salary e. $2000 dividend income f. $500 partnership income

A, D, E, F

Which of the following statements are correct when describing "Workers' Compensation?" a. Workers compensation benefits are not taxable to the recipient because the payments result from a physical injury b. Workers' compensation benefits are taxable to the recipient because this insurance was provided tax-free by the employer c. Both "worker's compensation" and "unemployment compensation" are taxable to the recipient d. Workers' compensation is another term for "unemployment compensation" e. Workers' compensation benefits are paid to employees who have been injured in a work-related situation

A, E

Certain types of investments may be tax-advantaged when used to help fund higher education. The growth in value and earnings from the investment are nontaxable if the proceeds are used to pay for qualifying educational expenditures. Which of the following types of investments qualify for this treatment? a. Section 529 plans b. U.S. Series EE bonds c. Coverdell savings account d. Municipal bonds e. Roth IRA plans

A/B/C

Which of the following choices describe exclusions and deferrals for tax purposes? a. These provisions are narrowly defined b. These provisions are the result of specific congressional action c. These provision reduce the tax liability dollar for dollar d. These provisions are often granted in order to subsidize or encourage particular behaviors e. These provisions are generally deducted from AGI and reduce taxable income

A/B/D

Which of the following represents economic benefits to a taxpayer? a. Cash received for completing a job b. Interest income on investments c. Cash received from a bank loan d. A computer received in exchange for services rendered

A/B/D

Which of the following cash receipts are taxable to the recipient? a. Gambling winnings b. Prizes c. Child support d. Loan Proceeds e. Awards

A/B/E

Chad has received the following income and benefits during the current year: $65,000 salary, $4,800 employer-provided health insurance, $1,500 municipal bond interest, $2,000 dividend income, $500 from a partnership, and a $10,000 judgement for lost wages due to an age discrimination lawsuit. What amounts should Chad include in his gross income? a. $2,000 dividend income b. $500 partnership income c. $4,800 health insurance d. $1,500 municipal bond interest e. $10,000 judgement f. $65,000 salary

A/B/E/F

Choose the type of investments that generate tax-exempt income: a. Educational savings plan b. U.S. Savings Bonds c. Municipal bonds d. Certificate of Deposit e. Life insurance policy

A/C/E

In which of the following situations has constructive receipt occurred in the earlier year? a. Arnold earned interest of $300 on his savings account, but did not withdraw the funds until the following year b. Andrew worked the last two weeks of the year, but he will not be paid for those hours until Jan 4 c. Alex was out of town on Dec 31 and, therefore, unable to pick up his final paycheck. He picked it up on Jan 2 d. Ashley's last paycheck of the year was dated Dec. 30. She was unable to cash the check because the employer had insufficient funds e. Amber received a year-end bonus check dated Dec 28, but she did not cash the check until Jan. 3

A/C/E

Which of the following choices are characteristics of Coverdell Educational Savings Accounts? a. The maximum yearly contributions to the account are limited to $2,000 for each beneficiary b. The maximum yearly contributions to the account are limited to $2,000 per taxpayer ($4,000 for mfj) c. Reasonable room and board costs are included in qualified higher education expenses d. Expenses for higher education, such as tuition and books, are NOT qualified educational expenses for this type of account e. The contribution limit is available to all taxpayers and is NOT subject to any phase out for higher levels of AGI f. The distributions can be made for qualified education expenses from kindergarten through 12th grade g. Earnings on the account are NOT taxable if used for qualified educational expenses

A/C/F/G

Which of the following situations will result in an award being excluded from gross income? a. When the award is a non cash item valued at less than $400, and given for either safety or years of service by an employee b. When the award has a charitable component, such as a television giveaway, where guests are given needed items c. When the award is given during the holidays, such as a Christmas cash bonus, since the intent of the award is to be a gift d. When the award is given for scientific, literary, or charitable achievement and meets certain other requirements

A/D

Brenda received a $15,000 college scholarship for the current year. She used the scholarship to pay the following fees: tuition $9,000 lab fees $60, parking fees $120, books $750, housing $4,000, and a meal plan $1,070. What can she exclude from gross income? a. $60 lab fee b. $4,000 housing c. $120 parking fees d. $1,070 meals e. $9,000 tuition f. $750 books

A/E/F

What is an "Above" and "Below" the line deduction?

After finding a deduction section and applying any limitations to the expenditure, it must be determined where to take the deduction -- either from gross income in computing adjusted gross income (aka, above the line), or from adjusted gross income in computing taxable income (aka, below the line deductions). The proverbial "line" is that which separates adjusted gross income from taxable income in the computation of federal tax liability.

The term used when one former spouse is required to provide financial support to the other spouse pursuant to a legal separation or divorce is ________.

Alimony

Alimony and Texas

Alimony not present in Texas, but they do have separate "maintenance" payments. The name doesn't matter, if you meet the requirements, the federal rules still treat as alimony.

In the case of a divorced couple, _____ payments are included in the gross income of the recipient and deductible _____ AGI for the payor.

Alimony/ For

gross

All income from whatever source derived is _____ income.

Which of the following statements about alimony payments is true for divorce agreements executed before 2019? To qualify as alimony, payments must be made in cash. Alimony payments are includible in the gross income of the recipient. To qualify as alimony, payments cannot continue after the death of the recipient. To qualify as alimony, payments must be made under a written agreement or divorce decree that does not designate the payments as "nonalimony" or child support. All of the choices are correct.

All of the choices are correct

Brenda has $15,000 in U.S. Series EE saving bonds and she is considering whether to cash in the bonds. Under what conditions can Brenda exclude the interest on the savings bonds from her gross income? Brenda can exclude the interest if she uses the proceeds to pay for college tuition. Brenda's modified AGI must be below a phase-out range for the exclusion. The proceeds must be used for higher education expenses of Brenda, her spouse, or Brenda's dependent. All of these are necessary conditions for Brenda to exclude the interest. None of these are correct - the interest is always included in gross income

All of these are necessary conditions for Brenda to exclude the interest.

Acme published a story about Paul and as a result Paul sued Acme for damage to his reputation, emotional distress, and punitive damages. Paul won an award of $20,000 for damages, $5,500 for emotional distress, and $50,000 for punitive damages. What amount must Paul include in his gross income?

All of these benefits are included in gross income.

What is the rule for expenses for the production of income (investment expenses)?

Allowed to deduct 1) all ordinary and necessary expenses 2) paid during the taxable year 3) for the production, collection, maintenance of the property, or in connection with the determination, collection or return of taxes. After 2017, for practical purposes, only those deductions associated with a rent or royalties will be deductible.

Alimony and gross income

Amount of the payment is included in the gross income of the person receiving it and is deductible for AGI by the person paying it. Thus, alimony shifts income from one spouse to the other.

If taxpayer receiving life annuity lives shorter than originally estimated

Amount of the unrecovered investment (the initial investment less the amounts received that is treated as a nontaxable return of capital) is deducted on the taxpayer's final income tax return

What is the constructive receipt doctrine?

An item is constructively received by a taxpayer when it is credited to his account, set apart for him, or otherwise made available so that he may draw on it at any time, but it isn't constructively received if its actual receipt is subject to substantial limitations or restrictions.

common law

Ann and Andy live in a state where all of the income earned from the services of Ann are included only in her gross income. they live in a state with a(n) _____ system.

What is the rule for an installment sale?

Any disposition of property where at least 1 payment is received after the close of the taxable year in which the disposition occurs. Exceptions: 1) no installment sales for dealers of either real or personal property 2) no installment sales for stocks sold on established market

What are the elements for a capital asset?

Any property held by the taxpayer which isn't 1) inventory, 2) depreciable trade or business property 3) real property used in a trade or business 4) accounts receivable 5) literary copyrights in author's possession 6) patents. Any property interest qualifies (real, personal, tangible, or intangible).

nontaxable

Any reimbursement a taxpayer receives from a medical or accident insurance policy for medical expenses paid by the taxpayer during the current year are _____ (taxable/nontaxable) for the taxpayer.

What is necessary?

Appropriate and helpful. Doesn't require necessity in the sense that its absolutely required for the business.

Prizes, Awards, and Gambling Winnings

Are all included in gross income.

The _______ ____ __________ doctrine holds that the taxpayer who earns income from services must recognize the income, and the income from property is taxed to the person who owns the property,

Assignment/Of/Income

Downside of the first exception of the Prizes, Awards, and Gambling Winnings rule

Award recipient does not actually get to receive/keep the cash from the award. However, for tax purposes it is more beneficial for the recipient to exclude the award from income entirely by immediately transferring it to a charitable organization than it is to receive the award, recognize the income, and then contribute funds to a charity for a charitable deduction.

First Exception to the Prizes, Awards, and Gambling Winnings rule

Awards for scientific, literary, or charitable achievement such as the Nobel Prize are excluded from gross income, but ONLY IF (1) the recipient was selected without any action on his part to enter the contest or proceeding, (2) the recipient is not required to render substantial future services as a condition to receive the prize or award, and (3) the payer of the prize or award transfers the prize or award to a federal, state, or local governmental unit or qualified charity such as a church, school, or charitable organization designated by the taxpayer.

Allison won't $3,200 betting on a long-shot in the Kentucky Derby. Unfortunately, she lost her lucky streak and ended up losing $5,000 on her next (and last) trip to the racetrack. Allison heard that she will be able to deduct her gambling losses, so she is feeling a little better. How much of Allison's losses will she be able to deduct? a. $0- Gambling is not a business venture, so she will not deduct any of her losses b. $3,200- She can only deduct the losses to the extent of her winnings c. $5,000- She can deduct the full amount if she has documentation that verifies the losses

B

How are the proceeds from a life insurance policy treated if the policy is cashed in early for its surrender value when there is no chronic or terminal illness present? a. The excess of the cash surrender value over the premiums is deducted from the taxpayer's gross income b. The excess of the cash surrender value over the premiums is included from the taxpayer's gross income c. The proceeds from the policy are excluded from the taxpayer's gross income d. The proceeds from the policy are fully included in the taxpayer's gross income

B

What is the typical tax treatment for income from labor such as salaries, wages, and fees? a. It is nontaxable b. It is taxable c. It is tax-deferred d. It is tax-exempt

B

Which of the following fringe benefits provided by an employer is NOT excluded from gross income? a. Medical insurance b. Group life insurance coverage in excess of $50,000 c. Qualified transportation expenses d. De minimis (or small) benefits

B

Which of the following options is NOT available to taxpayers who have worked outside the United States and meet the requirements necessary to receive tax relief on their foreign earnings? a. Income exclusion for foreign-earned income b. Tax deduction for foreign-earned income c. Tax deduction for foreign taxes paid d. tax credit for foreign taxes paid

B

Which of the following statements is correct concerning a gift? a. A gift may be subject to "gift tax" which is paid by the person receiving the gift b. A gift may be subject to "gift tax" which is paid by the person giving the gift c. A gift is included in the gross income of the person receiving the gift and subject to income tax d. A gift is included in the gross income of the person giving the gift and subject to income tax

B

Which of the following statements is correct regarding the recognition of income? a. Income is realized when a taxpayer receives a refund of an amount that was not deducted in a prior period b. Income may be in the form of cash, property, or services received in a transaction c. Income is realized for the return of capital received in a sales transaction

B

Which of the following statements is incorrect regarding gross income? a. Exclusions and deferrals are a result of specific Congressional action and narrowly defined b. Income is only included in gross income when spelled out in specific tax provisions c. Some types of income may be temporarily deferred from gross income to be included in a later year d. Specific types of income may be excluded from gross income by Congress

B

Which of the following types of imputed income are not included in the gross income and are not taxable to the person receiving the benefit? a. An employer's $12000 loan to an employee with no interest on the note b. A bargain purchase between a father and his son c. Employee discounts of 25% on services

B

Which of the following statements are characteristics of alimony in a divorce decree entered into before 2019? a. Alimony can include property divisions between ex-spouses in a divorce agreement b. Alimony is included in the gross income of the person receiving it c. Alimony payments can not continue after the death of the recipient d. Alimony is deductible for AGI for the person paying it e. Alimony is not deductible to the person paying it f. Alimony may not be paid to a spouse who is still living with the payor as long as there is a separation agreement in place g. Alimony must be paid in cash

B, C, D, G

Under the realization principle, what must occur for income to be realized? a. A taxpayer must receive cash to realize income b. A taxpayer must engage in a transaction with another party c. A measurable change in wealth must occur, regardless of whether a transaction takes place d. The transaction must result in a measurable change in property rights

B/D

Choose the types of investments that generate taxable income: a. Educational savings plan b. Corporate bonds c. Municipal bonds d. U.S. Savings Bonds e. Certificate of Deposits

B/D/E

Mary has received the following income and fringe benefits during the current year: a $73,000 salary; $600 in employer-provided disability premiums; $5,000 in workers; compensation; $1,200 in corporate bond interest; a $23,000 car won on a game show; and a $1,200 health insurance reimbursement for medical expenses paid during the year. Which of these amounts should Mary include in her gross income? a. $1,200 health insurance reimbursement b. $1,200 corporate bond interest c. $5,000 workers' compensation d. $600 disability premiums e. $73,000 salary

B/E

Imputed income examples

Bargain purchases and below-market loans.

Income from Flow-through Entities cont. (report on tax return)

Because different types of income and deductions may be treated differently for tax purposes (e.g., qualified dividends are eligible for a special low tax rate), each item the partners or shareholders report on their tax returns retains its underlying tax characteristics. That is, the partners are treated as if they personally received their share of each item of the flow-through entity's income.

Thi year bill purchased 1K shares of cain cs for 12 per share. T year end the cain shares were worth $32 per share. What amount must bill include in income this year?

Bill has not realized any gain. It is not realized until it is sold.

Define qualified residence interest.

Both acquisition indebtedness and home equity indebtedness are treated as qualified residence interest.

Business, investment and personal expenses are deducted when?

Business interest expense is a deduction for AGI, investment interest expense is an itemized deduction subject to limitations, and personal interest is generally not deductible

What are the limitations on business deductions?

Business may not deduct fines paid to a government for violation of a law, such as fines for polluting or fines for violating highway safety laws such as driving trucks with load in excess of the maximum weight allowed. Entertainment expenses: Starting in 2018 entertainment expenses are not deductible.

Chester incurred $14,500 in hospital and medical bills during the current year. His health insurance policy reimbursed him $11,600 toward those expenses. What amount should be included in Chester's gross income? a. $2,900 b. $11,600 c. $0 d. $14,500

C

What are the tax consequences for a taxpayer who dies before recovering his investment in an annuity contract? a. One-half of the annuity payment is taxable on the taxpayer's final income tax return b. The entire amount of the annuity received in the year of death is nontaxable on the taxpayer's final income tax return c. The amount of the unrecovered investment is deducted on the taxpayer's final income tax return d. The entire amount of the annuity received in the year of death is taxable on the taxpayer's final income tax return

C

Which of the following statements is CORRECT regarding the recognition of income? a. Income is realized for the return of the capital received in a sales transaction b. Income is realized when a taxpayer receives a refund of an amount that was not deducted in a prior period c. Income may be in the form of cash, property, or services in a transaction

C

Which of the following statements is NOT correct regarding alimony? a. Alimony is deductible for AGI for the person paying it b. Alimony payments must be made in cash c. Alimony is another term for child support d. Alimony is included in gross income of the person receiving it

C

Which of the following types of interest income is excluded from federal taxation? a. Federal government bonds b. Corporate bonds c. Municipal bonds

C

During the current year, Sam received interest income from the following investments: $400 from State of Wyoming bonds, $200 from Ford Motor Co., $50 from City of Laramie bonds, $100 from U.S. Treasury bonds. How much of the interest received will be included in gross income? a. $200 b. $700 c. $300 d. $600

C The State of Wyoming and City of Laramie bonds are not taxable.

Mary has received the following income and fringe benefits during the current year: a $73000 salary, $600 premiums paid by the employer-provided disability insurance, $5000 in workers' compensation, $1200 health insurance reimbursement for medical expenses paid during the year. Which of these amounts should Mary include in her gross income? a. $5000 workers' compensation b. $600 disability premiums c. $1200 corporate bond interest d. $73000 salary e. $1200 health insurance reimbursement

C, D

What are the advantages of the realization principle for defining gross income? a. It allows the taxpayer to exclude the transaction from gross income b. It provides the taxpayer with the opportunity to shift income to a related taxpayer who is in a lower tax bracket c. It provides the taxpayer with the wherewithal to pay when cash is received in the transaction d. It provides an objective measure of the value of the transaction

C/D

Which of the following statements are CORRECT when describing "Workers' Compensation?" a. Both "worker's compensation" and "unemployment compensation" are taxable to the recipient b. Workers' compensation is another term for "unemployment compensation" c. Workers' compensation benefits are not taxable to the recipient because the payments result from a physical injury d. Workers' compensation benefits are paid to employees who have been inured in a work-related situation e. Workers' compensation benefits are taxable to the recipient because this insurance was provided tax-free by the employer

C/D

Which of the following rules must be met for a taxpayer to be able to exclude the gain on the sale of a personal residence? a. The taxpayer must NOT have used the gain exclusion provision in the five years prior to the sale b. The taxpayer may use the exclusion on a vacation home or second home if NOT used for the principal residence c. The exclusion is $500,000 for taxpayers who are married filing jointly d. The taxpayer must have owned the residence for at least two years of the five year period prior to the sale e. The taxpayer must have used the property as a personal residence for a total of two or more years during the five year period prior to the sale

C/D/E

Which of the following types of income are generated from property ownership? a. Wages earned by an electrician b. Salary earned by an accountant c. Gain from the sale of a building d. Rental income from lessees e. Dividends received on corporate stock f. Interest earned on U.S. Treasury bonds

C/D/E/F

Non-fixed annuities- expected value

Calculated as number of annual payments from the table (referred to as the expected return multiple) is multiplied by the annual payment amount.

In general, when a taxpayer cashes out a life insurance policy before death, taxable income may result. However, if the taxpayer is _________ ill, the portion of the income required for long-term care is excluded from gross income. If the taxpayer is ________ ill, the proceeds are NOT taxable.

Chronically/ Terminally

Who computes the taxpayer's income when there is no accounting method available?

Commissioner may compute taxpayer's income. If no method of accounting has been used or if the method used by the taxpayer doesn't accurately reflect income, the service may compute the taxpayer's income in any reasonable manner.

Ann and Andy live in a state where all of the income earned from the services of Ann are included only in her gross income. They live in a state with a _____ system.

Common law

What is the rule for standard deductions?

Compare the standard deduction amount to a taxpayer's itemized deduction and take which is larger. The standard deduction is made up of two parts: the basic and additional standard deductions. Amounts almost doubled by Tax Cuts and Jobs Act of 2017.

Earned income

Compensation and other forms of income received for providing goods and services in the ordinary course of business.

How do you compute depreciation for real property?

Compute with straight-line method and longer recovery periods than for tangible property.

_____ is deemed to occur when the income has been credited to the taxpayer;s account or when the income is unconditionally available to the taxpayer, the taxpayer is aware of the availability and there are no restrictions on the income.

Constructive Receipt

What is cost basis?

Cost basis is the amount of money used to purchase the property. This includes any liabilities incurred in purchasing the property.

alimony

Court-ordered cash payments pursuant to a divorce or legal separation which provide financial support to an ex-spouse and do not continue after the death of the ex-spouse are referred to as:

tax advantaged account that can be used to fund qualified educational expenses from kindergarten through 12th grade and higher education expenses such as tuition, books, fees, supplies, and reasonable room and board

Coverdell Educational Savings Account

What are credits?

Credits are dollar-for-dollar reduction of tax liability.

Abby sold a parcel of land for $18,000. She paid a real estate agent a commission of $1,200 for assisting with the sale. Abby had purchased the land several years earlier for $14,500. What is the gain on the sale of the land? a. Gain of $3,500 b. Gain of $16,800 c. Gain of $18,000 d. Gain of $2,300

D

Andrew earned $5,000 in wages while working a part-time job during the current year. He also received a scholarship for $12,000. He used $9,000 for tuition and $800 for books. The remaining $2,200 went to help cover the cost of housing. During the summer Andrew's uncle died and he inherited $30,000. What is the amount of Andrew's gross income for the current year? a. $14,800 b. $35,000 c. $44,800 d. $7,200 e. $36,200 f. $17,000

D

Bobby received a $3,500 scholarship for the semester. He used $3,000 to pay tuition to the community college, and the remaining $500 was paid toward textbooks. How much of the scholarship must be included in gross income? a. $3,500 b. $500 c. $3,000 d. $0

D

Janice and Jarrod are married and live in a community property state. Janice is NOT employed outside the home, and Jarrod earns a salary of $98,000. During the year, they earned $2,000 on investments that are owned jointly. The investments were made after they were married, with money earned by Jarrod. How is the gross income treated for federal income tax purposes? a. Jarrod is deemed to have earned $99,000 and Janice is deemed to have earned $1,000 b. Jarrod is deemd to have earned $98,000 and Janice is deemed to have earned $2,000 c. Jarrod is deemd to have earned $100,000 and Janice has no income d. Jarrod is deemd to have earned $50,000 and Janice is deemed to have earned $50,000

D

Which of the following fringe benefits provided by an employer is not excluded from gross income? a. Qualified transportation expenses b. Medical insurance c. De minimis (or small) benefits d. Group life insurance coverage in excess of $50,000

D

Which of the following situations satisfies the ownership test for excluding the gain on a personal residence? a. The taxpayer must have owned the residence for the two years prior to the sale b. The taxpayer must have owned the residence for more than 12 months c. The taxpayer must have owned the residence for five consecutive years prior to the sale d. The taxpayer must have owned the residence for two or more years during the past five-year period ending on the date of sale

D

Which of the following statements is correct concerning a gift? a. A gift is included in the gross income of the person giving the gift b. A gift is included in the gross income of the person receiving the gift c. A gift may be subject to "gift tax" which is paid by the person receiving the gift d. A gift may be subject to "gift tax" which is paid by the person giving the gift

D

Which of the following statements is not correct regarding alimony for divorce decrees before 1/1/19? a. Alimony is not deductible for AGI for the person paying it b. Alimony payments must be made in cash c. Alimony is included in gross income of the person receiving it d. Alimony is another term for child support

D

Which of the following types of cash receipts is NOT taxable to the recipient? a. Lottery winnings b. Gambling winnings c. Raffle prizes d. Gift from a friend

D

In order to exclude the maximum amount of foreign-earned income form U.S. taxation, the following conditions must be met. a. The taxpayer must have lived in the country for at least 6 months in the current year b. The Taxpayer must NOT be a U.S. citizen c. The taxpayer must be an employee of the U.S. government on a temporary assignment in the foreign country d. The taxpayer must be considered a "resident" of the foreign country e. The taxpayer must have resided in the foreign country for 330 days in a consecutive 12-month period

D/E

Which of the following choices are characteristics of qualified tuition programs, also known as Section 529 plans? a. The distributions can be made for qualified education expenses from kindergarten through 12th grade b. The maximum yearly contributions to the account are limited to $2,000 for each beneficiary c. Distributions to contributors are NOT subject to income tax, but they do incur a 10% penalty d. Earnings on the account are NOT taxable if used for qualifying higher education expenses e. Distributions made to the beneficiary for purposes other than education will incur taxation and a penalty on the earnings of the plan

D/E

What is the rule for student loan interest?

Deduction allowed: 1) Indebtedness incurred by the taxpayer 2) Paid during the year 3) For higher education expenses 4) Deduction limited to $2,500 5) Deduction phases out in 2018 for taxpayer with AGI's over $65,000 ($135,000 for married couples filing a joint return).

Rather than claiming the foreign-earned income exclusion, taxpayers may claim a foreign tax _______ or a foreign tax ________ for income taxes paid to other countries.

Deduction/ Credit

What is cost depletion?

Deductions are allowed under cost depletion by taking into account the adjusted basis of the property, the number of units of the resource sold within a taxable year, and the number of units of the resource estimated to remain at the end of the taxable year.

The rate at which taxpayers are taxed on gains from property dispositions and the extent to which they can deduct losses from property dispositions depends on what?

Depends on whether the taxpayer used the asset for business purposes, investment purposes, or personal purposes.

Tax treament of unearned income

Depends upon the type of income and, in some circumstances, the type of the transaction generating the income. See textbook for problems on income from property.

How do you compute depreciation for tangible property?

Depreciate the basis of the property using the appropriate 1) depreciation method 2) recovery period and 3) applicable convention. Till 2023 have 100% deductibility for most items.

What is the general rule for depreciation?

Depreciation deductions allow if 1) property suffers from exhaustion, wear and tear and 2) either used in a) trade or business or b) held from the production of income (investment property). Till 2023 have 100% deductibility.

To provide tax relief for insolvent taxpayers:

Discharge of Indebtedness is not taxable if taxpayer is insolvent before/after debt forgiveness.

Bargain purchases general rules

Discount received on the purchase from a related party is taxable, but the tax consequences vary based on the relationship of the parties (compensation income, taxable dividend and gifts).

C Corporation Taxation - Distributions

Distributions: Distributions from a corporation are treated either as 1) dividends (to the extent the distribution was out of the corporation's earnings and profits), 2) a return of basis (no income), or 3) gain.

political

Donations to _____ organizations are not deductible for federal income tax purposes.

Income from services, including business income, is referred to as _______ income.

Earned

education

Earnings on investments in plans such as section 529 plans, Coverdell savings accounts, and U.S. series EE bonds are excluded from taxation if the proceeds are used for qualifying _____ expenses.

What is the general rule for gross income?

Except as otherwise provided, gross income includes all income from whatever source derived.

What is insolvent?

Excess of taxpayer's liabilities less the FMV of taxpayer's assets.

Nonrecognition provisions refer to specific types of income that taxpayers realize but are allowed to permanently _______ from gross income or temporarily ________ until a later period.

Exclude/Defer

for

Expenses associated with generating rental or royalty income are deductible _____ AGI.

A taxpayer who receives money when taking out a bank loan will include the amount borrowed in their gross income under the all-inclusive definition of income. T/F

F

Gambling winnings are excluded from gross income. T/F

F

Worker's compensation benefits received from a state-sponsored workers' compensation plan are taxable. T/F

F

T/F In general, prizes awarded to taxpayers are excluded from gross income

FALSE

T/F Scholarships received by college students qualify as gifts and are, therefore, nontaxable. The actual use of the money (tuition, fees, housing, meals, and any other expenses) does not affect the taxable status of the scholarship

FALSE

Gambling winnings are excluded from gross income.TF

False

T or F: Gross income only includes income received in cash because cash is a measurable change in wealth.

False

Workers compensation benefits received from a state-sponsored workers' compensation plan are taxable.FT

False

What is recharacterization?

First characterize the gain under Section 1245 (depreciable non-real property) and Section 1250 (depreciable real property). any remaining gain (and all the losses) will probably be characterized by Section 1231.

How do you determine if interest is deductible?

Follow what the taxpayer does with the borrowed funds. Interest expense may be deductible if the loan is used for business purposes, to purchase investments, to purchase a principal residence and up to one vacation home, or to pay for certain educational expenses.

What is the rule for alimony through the end of 2018?

For a payment to be alimony it must be: 1) cash 2) received by, or on behalf of a spouse, 3) pursuant to a written divorce or separation agreement 4) an agreement that doesn't say its not alimony 5) not members of the same household 6) without liability after death of the payee spouse 7) not for child support.

When analyzing gains from property, explain Step 1 determining gains (or losses) realized.

For all "sales or other dispositions of property" you must determine the amount of gain realized. This is computed by taking the amount realized less the adjusted basis. Sale or other disposition of property: Generally, except for a pure gift situation, a taxpayer must compute gain (or loss). This is because there is a "realization event."

What is the rule for installment method?

For any 1) installment sale 2) the taxpayer may use the installment method to recognize gain and 3) may elect out and have all gain recognized in the year of sale. Must recognize gain over the period that receive payment for property. Done by multiplying the payment by the gain realized divided by the total contract price.

What is the general rule for the inclusion of alimony and separate maintenance payments?

For any divorce or separation agreement executed after Dec. 31, 2018, alimony and separate maintenance payments aren't included in the income of the payee spouse and aren't deductible by the payor spouse.

What is the rule for alimony after 2018?

For divorce or separation instruments executed after December 31, 2018, alimony payments aren't deductible. Rule of non-deductibility doesn't apply to those who are currently paying alimony. New rule can apply to pre-2019 divorce or separation instruments if 1) they're modified after December 31, 2018, and 2) they expressly provide the new rules of non-deductibility apply.

Second Exception to the Prizes, Awards, and Gambling Winnings rule

For employee awards for length of service or safety achievement. These nontaxable awards are limited to $400 of tangible property other than cash per employee per year. The award is not excluded from the employee's income if circumstances suggest it is disguised compensation.

375000

For mortgages obtained in 2018, homeowners with a filing status of married filing separately may deduct mortgage interest on up to _____ of acquisition indebtedness.

What is the general rule for capital losses?

For non-corporate taxpayers, capital losses can only be deducted up to the amount of capital gain plus an extra $3,000 (or, if less, the losses over the gains).

What is the deduction for taxes rule?

For tax years from 2018 to 2025 a deduction is allowed for 1) state and local real property taxes 2) state and local personal property taxes 3) state income taxes and 4) state sales tax. The deduction for individuals is limited to $10,000 ($5,000 married filing separately). Federal taxes are NOT deductible. Neither federal income taxes, nor the employee's share of Social Security taxes are deductible.

What is a qualified trade or business?

For taxable years from 2018 through 2025, non-corporate taxpayers can take a deduction of up to 20% on qualified business income. This deduction is new in 2018 and is designed to reduce the tax rate on non-corporate business income. The deduction can be used regardless of whether a taxpayer itemizes or uses the standard deduction.

C Corporation Taxation - Formation

Formation (Transfers to a Corporation Controlled by Transferor Section 351): No gain or loss shall be recognized if 1) property transferred to a corporatoin 2) in exchange for stock of such corporation 3) and immediately after the exchange such person (or persons) has "control" (80% of more ownership of the corporation's stock in vote and number of shares).

Below-market loans indirect economic benefit

Function of the amount of the loan and the difference in the market interest rate and the rate actually charged by the lender

What is a Fringe Benefit?

Generally finge benefits are treated as gross income to the employee. Certain fringe benefit are excluded for policy or administrative reasons from the employee's gross income. These include: 1) No-additional-cost services 2) Qualified employee discounts 3) Working condition fringe 4) De minimis fringe 5) Qualified transportation fringe 6) Qualified retirement planning services

How do you compute depreciation for intangible property?

Generally intangible property can't be amortized (what depreciation is called when taken on an intangible asset), but certain "Section 197 intangibles" are allowed to be amortized over a 15-year period.

What is the rule for accrual method?

Generally, the a/m is required to be used by business taxpayers of any significant size. Income: Under the accrual method an item of income is incredible when 1) all events have occurred that fix the right to receive it, and 2) the amount can be determined with reasonable accuracy. Deductions: An expense is deductible when 1) all events have occurred that fix the liability, and 2) the amount can be determined with reasonable accuracy 3) but not sooner than when economic performance occurs. This means that a taxpayer can't deduct estimated future expenses no matter how accurate her estimation may be, because her liability for specific expenses can't be established.

C Corporation Taxation - Generally

Generally: Regular corporations (called "C corporations") are separate taxpayers and pay tax on taxable income, which is computed in a manner similar to the rules for individuals. Dividend distributions are taxable to individual shareholders and rates on dividends are the same as on long-term capital gains, up to 20%. Thus, earnings of a C corporation are subjected to double tax.

Which of the following types of cash receipts is not taxable to the recipient?

Gift from a friend

When an individual transfers property to another taxpayer during their life, without receiving or expecting to receive value in return, the property transferred is a _____ and is not included in the _____ (income/property) by the person receiving the property.

Gift, nontaxable

When an individual transfers property to another taxpayer during their life, w/o receiving or expecting to receive value in return, the property transferred is a(n) ________ and is __________ (taxable/nontaxable) to the person receiving the property.

Gift; Nontaxable

What is the general rule for personal injury?

Gross incoe doesn't include 1) damages received whether by suit or agreement and whether as lump sums or periodic payments 2) on account of personal physical injury or physical sickness, 3) other than punitive damages

What is the general rule for excluding life insurance from gross income?

Gross income doesn't include 1) amounts received under a life insurance contract, 2) if such amounts are paid by reason of the death of the insured, 3) unless transferred for valuable consideration or instead of a lump sum payment, the payments are prorated over time.

What is the general rule for the exclusion of gifts, bequest, devise, or inheritance?

Gross income doesn't include property or services recieved by gift, bequest, devise, or inheritance.

Harold receives a life annuity from his qualified pension that pays him $5,000 per year for as long as he lives. Later this year Harold will recover the remainder of his cost of the annuity. Which of the following correctly describes how the annuity payments are taxed after Harold has recovered the cost of the annuity? Harold will include the entire amount of each annuity payment in gross income after he recovers the cost of the annuity. The entire amount of each annuity payment is excluded from gross income after Harold recovers his cost of the annuity.

Harold will include the entire amount of each annuity payment in gross income after he recovers THE COST of the annuity.

What is the hope credit?

Hope Education Credit (currently called the "American Opportunity Tax Credit") Up to $2500 credit in 2018 for 1) 1st 4 years of post-secondary education 2) student not convicted of felony drug offense 3) phased-out for taxpayer with AGI over $80k (singles)/$160k (joint return) Can have multiple Hope credits on one return. If married must file a joint return.

five years

How many years can excess charitable contributions be carried over before expiring?

Challenge from a tax perspective with annuities

How much of each annuity payment represents gross income (income taxed at ordinary tax rates) and how much represents a nontaxable return of capital (return of the original investment).

long term care

If a taxpayer cashes out a life insurance policy before death due to a chronic illness, she may exclude from income the amount used to pay for her _____.

What is the general rule for child support?

If any portion of the payment is fixed by the decree or agreement as being for support of the payor's children, the portion isn't deductible by the payor and isn't includible by the recipient. Any reduction in payment due to a contingency relating to a child (such as reaching the age of majority) are treated as child support rather than alimony. Payments are first allocated to non-deductible child support and then the alimony.

What is a non-recognition section?

If the gain or loss isn't recognized then you no longer need to worry about income. Will have issues on basis/holding period.

For the exclusion of life insurance from gross income, what is the rule in regard to payments of life insurance proceeds at a date later than death?

If the life insurance proceeds are paid out over time, the amount excluded equals the excluded amount divided by the beneficiary's life expectancy.

community property

In a(n) system, the income earned from services by one spouse is treated as though it was earned equally by both spouses.

fringe benefits

In addition to receiving a salary from a company, many employers provide _____ that are excluded from gross income.

nontaxable

In general, life insurance proceeds are _____ (taxable/nontaxable) to the beneficiary of the policy.

physical

In personal injury cases, any damages awarded due to _____ injury are exempt from taxation.

Define itemized deduction.

In the tax code the presumption is that expenditures aren't deductible and require a specific code section allowing for a reduction in either adjusted gross income or taxable income. The following are some of the most testable itemized deductions. Deductions save tax liability by a taxpayer's marginal rate.

In general, when a taxpayer's debt is discharged by a lender, the taxpayer's gross income will _____ the amount forgiven.

Include

Other types of payments that do not qualify as alimony

Include (1) property divisions (e.g., who gets the car, house) and (2) child support payments fixed by the divorce or separation agreement.

What is the general rule for the inclusion of employer provided group term life insurance?

Include employer provided group-term life insurance except for the costs of the first $50,000 of insurance. Must not discriminate as to key employees.

Taxpayers who realize an economic benefit must ______ the benefit in gross income unless it is specifically ______ by the tax code.

Include/ Excluded

property

Income from _____ takes different forms, such as dividends, interest, rents, royalties, and annuities.

unearned

Income from property is referred to as (earned/unearned) _____ income.

What is the rule for assigning income from property?

Income from property is taxable to the person who owns the property.

What is the rule for assigning income from services?

Income from services in includible in the gross income of the person performing the services. An exception exists if a person 1) makes a complete disclaimer 2) before services are performed. Community Property: by state statute the income from services of one spouse are treated as 1/2 property interest in each spouse.

What is the rule in regard to exclusion of discharge of indebtedness?

Income from the discharge of indebtedness can be excluded from gross income in case of 1) insolvency (to the amount insolvent), 2) Title 11 (bankruptcy 3) gift 4) purchase price adjustments and 5) certain student loans.

What is the rule for cash method accounting?

Income is recognized in the year the cash (or cash equivalent) is received. Deductions are taken in the year when paid, unless prepaid. Then they are taken when economic performance occurs.

For below-market-loans, the discounted interest rate is treated as interest _____ to the lender and interest _____ to the borrower.

Income/Expense

In general, when a taxpayer's debt is discharged by a lender, the taxpayer's gross income will _____ the amount forgiven.

Increase

Define Investment Interest.

Interest incurred to purchase investment property is deductible, but only up to the amount of net investment income included in gross income in the year.

Define interest.

Interest is compensation for the use or forbearance of money. Basically, "rent." The expenditure doesn't have to be labeled "interest" in order to qualify for the deduction (e.g., points).

Annuity definition

Investment that pays a stream of equal payments over time.

1300

Jamarcus injured his hand playing softball one weekend. The injury prevented him from being able to work. Luckily for Jamarcus, his employer pays the disability insurance premium and provides disability insurance as a nontaxable fringe benefit to all employees. Jamarcus received $1,300 in disability benefits wile he was away from work. Jamarcus should include ___ in gross income.

Janice and Jarrod are married and live in a community property state. Janice is not employed outside the home, and Jarrod earns a salary of $98000. During the year, they earned $2000 on investments that are owned jointly. The investments were made after they were married, with money earned by Jarrod. How is the gross income treated for federal income tax purposes?

Janice is deemed to have earned $50000 and Jarrod is deemed to have earned $50000

Joyce's employer loaned her $50,000 this year (interest-free) to buy a new car. If the federal interest rate was 3%, which of the following is correct?

Joyce recognizes $1,500 of imputed compensation income -employees recognize compensation income on below market loans from employers calculated using the federal interest rate

Capital Gains Rates/Ordinary Income

Long-term capital gain is taxed at a maximum rate of 20% (generally 15%), as opposed to ordinary income which is taxed at a maximum rate of 37%. To qualify for long-term capital gain, you must have a sale or exchange of a capital asset held for more than one year. If your income is lower enough such that you pay a maximum of 12% on ordinary income, then you would pay 0% on long-term capital gains. Short-term capital gain is taxed at a maximum rate of 37% which is the same as ordinary income. Short-term capital gain is recognized when you have held the capital asset for 12 months or less.

If a taxpayer cashes out a life insurance policy before death due to a chronic illness, she may exclude from income the amount used to pay for her ______- ________ _________.

Long/Term/Care

What is a gift?

Looking at the transferor's intent, was the transfer made with "detached and disinterested generosity." Link: Generally, the "adjusted basis" of a gift is a transferred basis. It is sometimes said that the transferee "steps into the shoes" of the transferor. The exception (the "except" clause) is with the gift of loss property (FMV < AB). In this case, if the property is later sold at a loss, the AB is the FMV at the time of transfer. Exception: No gifts for employer-to-employee transfers.

Bart sold a parcel of land for $21000. He paid a real estate agent a commission for $1500 fir assisting with the sale. Bart had purchased the land several years earlier fro $20000. What is the gain or loss on the sale of the land>

Loss of $500

What is the general rule for gambling losses?

Losses from wagering transactions are allowed only to the extent of the gain from such transactions. From 2018 to 2025 the limitation applies to expenses incurred in connection with the conduct of that individual's gambling activity (e.g., traveling to/from casino).

What is the rule in regard to interest?

May deduct 1) all interest, 2) paid or accrued during the taxable year, 3) unless "personal interest" Qualified residence interest and interest on educational loans are not "personal interest." Qualified residence interest and interest on educatoinal loans are not "personal interest" and are therefore deductible.

What is the lifetime learning credit?

May take credit for 20% of qualified tuition and relation expenses up to $10,000 ($2,000 maximum credit). If married must file a joint return. Phased-out in 2018 for taxpayers with AGI over $57k (singles) and $114k (joint return).

8160

Mitchell and Midge are married and file a joint return. Mitchell receives $9,600 in Social Security each year. Their modified AGI is $48,000. _____ of the Social Security benefits is subject to taxation.

Income from services

Most common; rarely exempt Salary, wages, fees taxpayer earns through services in nonemployee capacity and unemployment compensation. often referred to as earned income- generated by efforts of taxpayer

Interest on _____ bonds is excluded from federal income taxation.

Municipal

What is the general rule for the deduction of losses?

Must be 1) a "loss" (realized/recognized from above) 2) that isn't compensated for by insurance or otherwise. Loss is limited to the adjusted basis of the property.

What are the special rules for travel?

Must be an ordinary and necessary expense and must be 1) away from home 2) in pursuit of a trade or business 3) not lavish or extravagant. Meals are only deductible if work requires sleep and rest (the "overnight rule").

What is a qualifying child?

Must have 1) proper relationship (child of the taxpayer or spouse (and adopted children), any descendants of the child, brother, sister, stepbrother, stepsister or any descendants of those individuals) 2) meet the age requirements a) under 19 or b) a full time student under 24 3) must reside with the taxpayer for more than 1/2 of the taxable year 4) child must not provide more than 1/2 of their own support and 5) hasn't filed a joint return with their spouse for the taxable year.

What is a qualifying relative?

Must have 1) proper relationship (children and their descendants, brother, sister, father, mother, niece, nephew, uncle, aunt, in-laws, and any individual (other than spouse) who share the same principal place of abode as the taxpayer) 2) make less gross income than exemption amount 3) the taxpayer must provide over 1/2 the support 4) isn't a qualifying child.

is mere appreciation in wealth (economic income) considered in realized income Y/N

N

What is home equity indebtedness?

No deduction of home equity indebtedness for tax years from 2018-2025.

What is the like-kind exchange rule in the recognized gain?

No gain or loss is recognized when 1) real property used in a trade or business or held for investment (other than stock or securities) 2) is exchanged solely for other real property of a "like kind" 3) which is to be held either for productive use in a trade or business or for investment

What is the recognized gain of property transferred between spouses and former spouses incident to divorce?

No gain recognized on these transfers.

This year Bill purchased 1,000 shares of Cain common stock for $12 per share. At year-end the Cain shares were worth $32 per share. What amount must Bill include in income this year?

No realization occurs until the stock is sold.

Is a final divorce decree necessary to have an alimony exemption?

No! Alimony can be present under a written separation agreement. A final divorce decree isn't required. the one rule that changes is under a written separation agreement that spouses CAN live in the same household.

Devon owns 1,000 shares of stock worth $10,000. This year he received 200 additional shares of this stock from a stock dividend. His 1,200 shares are now worth $12,500. Must Devon include the dividend paid in stock in income? No, dividends paid in stock are generally not included in gross income. Yes, dividends paid in stock are generally included in gross income.

No, dividends paid in stock are generally not included in gross income.

This year Barney purchased 500 shares of Bell common stock for $20 per share. At year-end the Bell shares were only worth $2 per share. What amount can Barney deduct as a loss this year?

None - Barney is not entitled to a loss deduction

This year Bill purchased 1,000 shares of Cain common stock for $12 per share. At year-end the Cain shares were worth $32 per share. What amount must Bill include in income this year? $12,000 $20,000 $32,000 Bill can deduct $12,000 None of the above - Bill has not realized any gain.

None of the above - Bill has not realized any gain.

Any reimbursement a taxpayer receives from a medical or accident insurance policy for medical expenses paid by the taxpayer during the current year are _____ for the taxpayer.

Nontaxable

In general, life insurance proceeds are ________ to the beneficiary of the policy.

Nontaxable

What is ordinary?

Of common or frequent occurrence in the type of business involved.

When is marital status determined?

On the last day of the taxpayer's taxable year. Therefore, if the spouse get divorced during the year neither may file a joint return for that year.

For the exclusion of life insurance from gross income, what is the rule in regard to transfers for valuable consideration?

On transfers for valuable consideration the exclusion is limited to the amount paid plus any other premiums paid by the buyer unless transfer 1) to her partner in a partnership, 2) her partnership, or 3) corporation in which she is a shareholder or an officer.

Annuity exclusion ratio

Original investment/ expected value of annuity = return of capital precentage (determines portion of each payment that's nontaxable return of capital)

What is a personal exemption?

Personal exemption allowed for 1) all "taxpayers" on a return (joint returns have 2 taxpayers - each spouse), and 2) any dependents 3) of the "exemption amount." The exemption amount is zero from 2018 to 2025.

In personal injury cases, any damages awarded due to _______ injury are exempt from taxation.

Physical

What is the general rule for the inclusion of prizes and awards?

Prizes and awards are included in gross income. Exceptions exist for certain transfers to charities, qualified scholarships and employee achievement awards.

Income from _____ takes different forms, such as dividends, interest, rents, royalties, and annuities

Property

Income from ______ takes different forms, such as dividends, interest, rents, royalties, and annuities.

Property

What is an expense?

Property used in a trade or business or in the production of income that has a useful life of more than a year may not be "expenses" (its cost deduction in the year of purchase). Instead, the taxpayer must "capitalize" the cost (i.e., add the cost to basis). The consequences of this depend on the type of asset.

Joint-life annuity

Provide payments over the lives of two people. Annual payments each year until taxpayer AND taxpayer's spouse passes away.

What is the rule for qualified business income?

Qualified business income is the net amount of income, gain, deductions, and losses from a qualified business.

Loss on a dispoal sale

Realized when the proceeds are less than the tax basis in the property. Because the return of capital principle generally applies only to the extent of the sale proceeds, a loss does not necessarily reduce the taxpayer's taxable income (unless loss is deductible).

If a transfer of property between spouses does not meet the definition of alimony?

Recipient of the transfer excludes the value of the transfer from income, and the person transferring the property is not allowed to deduct the value of the property transferred.

Modified AGI

Regular AGI (excluding Social Security benefits) plus tax-exempt interest income, excluded foreign income and certain other deductions for AGI.

Identify the rule that determine whether a married taxpayer must recognize income earned by their spouse: -Residence of the married couple in a community property law state. -Tax benefit rule. -Residence of the married couple in a community property law state and tax benefit rule. -None of the above.

Residence of the married couple in a community property law state.

What are the two realizing events?

Sale: Conversion of property into cash Exchange: Transfer of one property from another

Formula for computing gain/loss from sale of an asset

Sales proceeds Less: Selling expenses =Amount realized Less: Basis (investment) in property sold =Gain (Loss) on sale

Sally is a cash basis taxpayer and a member of the Valley Barter club. This year Sally provided 100 hours of sewing services to the barter club in exchange for two football playoff tickets. Which of the following is a true statement?

Sally is taxed on the value of the football tickets even if she cannot attend the game

To calculate the amount realized on the sale of an asset, the proceeds is reduced by which of the following?

Selling Expenses

To calculate the amount realized on the sale of an asset, the proceeds is reduced by which of the following? Tax basis of the property. Selling expenses. Gain realized. Tax basis of the property and selling expenses. All of the choices are correct.

Selling expenses.

What are the limitations on the exclusion of gain?

Single Person: $250,000 Married, filing joint return: $500,000

250000

Single taxpayers meeting certain home ownership and use requirements can permanently exclude up to _____ of the realized gain on the sale of their principal residence.

Up to 85% of _____ benefits, in retirement, may ne taxable for taxpayers with moderate to high taxable income.

Social Security

Up to 85% of ___________ _____________ benefits, in retirement, may be taxable for taxpayers with moderate to high taxable income.

Social Security

Up to 85% of ________ ________ benefits, in retirement, may be taxable for taxpayers with moderate to high taxable income.

Social/Security

What is the basis in property transferred between spouses and former spouses if "incident to divorce" [Property Settlements]

Straight transferred basis Incident to divorce: Transfers between former spouses are "incident to divorce" if 1) done within 1 year of the cessation of the marriage or 2) related to the cessation of marriage. A transfer is pursuant to a divorce or separation instrument and it occurs within 6 years after the date on which the marriage ceases.

S Corporation and Partnership (Including Limited Liability Companies)

Subchapter S corporations and Partnership are not taxable entities. Instead, their individual owners will recognize taxable income in the same year that the entity recognizes the income, even though no cash distribution is made. Hence they're sometimes called "pass through" entities.

Generally, a portion of each payment from a purchased annuity represents a return of capital. T/F

T

Gross income includes all realized income that is recognized during the year. T/F

T

Larry received $4,250 from disability insurance that he purchased earlier this year from an insurance provider. Larry is allowed to exclude the $4,250 from his gross income. T/F

T

The cash method of accounting requires taxpayers to recognize income when they receive it in the form of cash, property, or services. T/F

T

The exclusion amount for a purchased fixed-term annuity can be calculated by dividing the cost of the annuity by the total number of payments. T/F

T

income received by the taxpayer's agent is considered to be received by the taxpayer T/F

T - also, a cash basis principal must recognize the income at the time it is received by the agent

T/F Income and deductions generated withing a partnership or S corp that are subject to various tax treatments (qualified dividends, capital gains, etc) retain their character when they flow-through to the owners rather than being reclassified as ordinary income or loss

TRUE

T/F income and deductions from a partnership or S corporation are taxed on the owners' tax returns rather than the entity tax return

TRUE

Identify the rule that determines whether a taxpayer must include in income a refund of an amount deducted in a previous year

Tax benefit rule.

Identify the rule that determines whether a taxpayer must include in income a refund of an amount deducted in a previous year: Tax benefit rule. Constructive receipt. Return of capital principle. Claim of right rule. None of the above.

Tax benefit rule.

To eliminate any tax advantages of below-market loans

Tax law generally requires the lender and borrower to treat the transaction as if: 1)The borrower paid the lender the difference between the applicable federal interest rate (compounded semiannually) and the actual interest paid (this difference is called imputed interest). 2)The lender then returned the imputed interest to the borrower.

What is a taxable year?

Tax liability is computed on a "taxable year". Most taxpayers have an accounting period (the time for which taxable income is determined) measured by the calendar year. However, a taxpayer may have a fiscal year accounting period ending on the last day of a month other than December.

Social Security Benefits for MFS:

Taxable Social Security benefits are the lesser of (a) 85 percent of the Social Security benefits or (b) 85 percent of the taxpayer's modified AGI + 50 percent of Social Security benefits.

Todd and Margo are seeking a divorce and no longer live together. Margo has offered to pay Todd $42,000 per year for five years if Margo receives sole title to the art collection. This collection cost them $100,000 but is now worth $360,000. All other property is to be divided equally. a. If Margo's payments cease in the event of Todd's death, how are the payments treated for tax purposes? Taxable to Todd and not deductible by Margo Not taxable to Todd but deductible by Margo Taxable to Todd and deductible by Margo Not taxable to Todd and not deductible by Margo

Taxable to Todd and deductible by Margo

What is the rationale behind like-kind exchanges in non-recognition?

Taxpayer has continued their ownership of the same type of assets.

If discharge of indebtedness makes taxpayer solvent?

Taxpayer recognizes gross income to the extent of his solvency.

Property Dispositions

Taxpayers can realize a gain or loss when disposing of an asset. Consistent with the return of capital principle we discussed before, taxpayers are allowed to recover their investment in property (tax basis) before they realize any gain.

What is a bequest, devise, or inheritance?

The AB of property received from a decedent is the FMV at the time of decedent's death.

What is the rule for an interest-free loan generally?

The Code will impute interest in certain situations where money is borrowed for low or zero interest. Treat as constrictive money paid from lender to borrower and then re-transferred from borrower to lender as interest.

claim of right

The _____ doctrine states that income has been realized if a taxpayer receives income and there are no restrictions on the taxpayer's use of the income.

realization

The _____ principle states that income is received when a taxpayer completes a transaction with another party that results in a measurable change in the property rights of the two parties.

What is adjusted basis?

The adjusted basis is the basis under the applicable section (based on how the property was acquired), adjusted for capital expenditures or depreciation. Conceptually, adjusted basis is the amount of capital (money) that a taxpayer has in any given piece of property.

What is loss limited to?

The adjusted basis of the property. From 2018 through 2025, casualty losses are limited to those from a Federally declared disaster.

What is amount realized?

The amount realized is the sum of money and the FMV of property other than money received. The amount realized includes any liability relieved.

What is the basic standard deduction?

The basic standard deduction amount for any taxpayer is based on their filing status. Married filing jointly/Surviving spouse: $24k Married filing separately: $12k Head of household: $18k Single: $12k

What is the basis in property acquired from a decedent?

The basis of property acquired from a decedent is the FMV at the date of decedent's death. If an alternate valuation date is used, the basis is the FMV on that date. The rule applies to both appreciated and depreciated property. Since property generally appreciates, this is called the "stepped-up basis" rule. Community Property Twist: Property which represents the surviving spouse's one-half share of community property held by the decedent and the surviving spouse under the community property laws of any State will also receive the FMV basis.

What are adjustments to basis?

The basis of the property may be increased (for capital expenditures/improvements made to the property) or decreased (for depreciation or losses). Think of basis as tracking the amount of capital a taxpayer has in any given piece of property.

What is percentage depletion?

The deduction is computed by multiplying the gross income from the property by a percentage which varies according to the type of mineral. The percentage depletion for oil is 15%. Large oil companies are only allowed to use cost depletion. Percentage depletion, which is generally more valuable since its not restricted to the actual cost of the oil properties, may be used by small, independent producers of oil.

investment income

The deduction of investment interest is limited to a taxpayer's net _____.

What is dissolution of marriage?

The dissolution of marriage is an area that covers multiple areas of taxation (gross income, deductions, adjusted basis, gain recognition, etc.).

529

The earnings on a section _____ plan are not taxable to the beneficiary if the distributions from the account are used to pay qualified higher education expenses.

How are the proceeds from a life insurance policy treated if the policy is cashed in early for its surrender value when there is no chronic or terminal illness present?

The excess of the cash surrender value over the premiums paid is included in the taxpayer's gross income

What is an inclusion section and what are the main three?

The following are items which are specifically included in gross income. While the definition of gross income in Section 61 would suffice, Congress wanted to make sure the following items were included. 1) Alimony and Separate Maintenance Payments (Section 71 - Repealed after 2018) 2) Prizes and Awards 3) Employer Provided Group Term Insurance

What is an exclusion section and what are the main three?

The general presumption for any "income" is that it is included in gross income. However, the rule states this is true "except as otherwise provided." Even though the following items are "income," Congress has provided statutory exceptions so they're never taken into account in computing tax liability, unlike a deduction which is subtracted out. 1) Life Insurance

What is a holding period?

The holding period starts on the date after purchase and ends on the date of sale. For stocks use the trade date to determine the holding period.

Andrews, Badin, and Carr formed a partnership, ABC. During Year 2, the partnership sold some land that was held for investment and generated a long-term capital gain. How will this income be reported on the partners' individual tax returns?

The income will retain its character and be reported as a long-term capital gain

What is a recourse loan?

The lender has a recourse against the borrower's other assets, not just the property purchased with the loan.

What is a non-recourse loan?

The lender's only recourse is against the security for the loan. If the borrower defaults, the lender can only take the security interest, not any of the borrower's other assets.

What is the rule for acquisition indebtedness?

The loan must be 1) incurred in acquiring, constructing, or substantially improving any qualified residence, 2) secured by such residence, 3) interest on up to $75,000 of principal. Qualified residence: A taxpayer can have two homes as qualified residences, one principal residence and one other residence. Acquisition indebtedness incurred on or before December 15, 2017 and after 2015 has a $1,000,000 limitation.

C Corporation Taxation - Earning and Profits

The measure of a corporation's economic capacity to make a shareholder distribution that isn't a return of capital. 1) is a measure of corporate wealth and utilized to distinguish a return of capital (recovery of a shareholder's investment) from a return on capital (or "income"). 2) The concept of e/p is sometimes compared to retained earning or taxable income.

Non-fixed annuities (payment over person's life)

The number of payments is uncertain. For these annuities, taxpayers must use IRS tables to determine the expected value based upon the taxpayer's life expectancy at the start of the annuity.

bunching

The system of shifting itemized deductions into one year such that the amount of itemized deductions exceeds the standard deduction for the year, and then deducting the standard deduction the next year is known as _____ itemized deductions.

What is the threshold amount for qualified business income?

The threshold amount is $157,500 for individuals ($315,000 for joint returns). When a taxpayer has taxable income (computed without the Section 199A deduction) over the threshold amount, certain lines of work (called a "specified service trade or business" in the statute) are phased-out from "qualified" status and additional limitations on the deduction occur are phased-in. The phase-outs and phase-ins are complete when taxable income is more than $50,000 ($100,000 for a joint return) over the threshold amount.

What is gift basis?

The transferee's adjusted basis is the same as the transferor's basis. It's said that the transferee "steps into the shoes" of the transferor.

What is the additional standard deduction amount?

There is an additional standard deduction of $1300 for taxpayers 65 or older and/or "blind" (20/200 vision or worse). These amounts increase to $1600 if the taxpayer is unmarried.

What is a trade or business deduction?

There shall be allowed as a deduction all 1) ordinary and necessary 2) expenses 3) paid or incurred during the taxable year 4) in carrying on 5) any trade or business.

Which of the following choices describe exclusions and deferrals for tax purposes?

These provisions are the result of specific congressional action. These provisions are narrowly defined. These provisions are often granted in order to subsidize or encourage particular behaviors.

When analyzing gains from property, explain Step 4 characterizing gains (or losses). (Generally)

This is the qualitative aspect to income/deductions. There is a rate preference as to capital gains from capital assets held for more than one year. Generally, the highest rate on long-term capital gain is 20%, while the highest rate on ordinary income is 37%.

Under the realization principle, income is realized when (1) a taxpayer engages in a _____ with another party, and (2) the _____ results in a _____ change in property rights.

Transaction, transaction, measurable

What is the cost basis for mortgage transactions?

Treat debt as equivalent to cash. When a person purchases property and borrows to finance the purchase, the amount of debt is included in the basis. When a person sells property, and the buyer assumes debt of the seller, it's treated like the buyer paid additional cash.

The deemed "payment" of the imputed interest in below-market loan transactions

Treated as interest income to the lender and interest expense to the borrower. The deductibility of the interest expense for the borrower depends on how she used the loan proceeds (for business, investment, or personal purposes).

Generally a portion of each payment from a purchased annuity represents a return of capital.TF

True

Generally, a portion of each payment from a purchased annuity represents a return of capital T or F

True

Gross income Includes all realized income that is recognized during the year. T/F

True

Larry Rceived 4,25 from disability insurance that he purchased earlier this year form an insurance provide. Larry is allowed to exclude the 4,250 from his gross income.TF

True

T or F: Income and deductions generated within a partnership or S corporations that are subject to various tax treatments retain their character when they flow-through to the owners rather than being reclassified as ordinary income or loss.

True

T or F: Punitive damaged are fully taxable to the recipient.

True

The cash method of accounting requires tax payers to recognize income when they receive it in the form of cash, property, or services.TF

True

The exclusion amount for a purchased fixed-term annuity can be calculated by dividing the cost of the annuity by the total number of payments.TF

True

false

True or false: Activities classified as hobbies can generate tax deductible losses that can be used against other types of income.

false

True or false: Business expenses are reported directly on the Form 1040 and are deducted for AGI.

false

True or false: Darlene owns stock in several different companies. when she received a dividend check from her avatar stock, she endorsed the checks and deposited the money in her daughter's checking account. consequently, her daughter will be assessed the tax on the dividends.

true

True or false: Gambling expenses and losses to the extent of gambling winnings are reported as miscellaneous itemized deductions.

true

True or false: Income and deductions generated within a partnership or S corporation that are subject to various tax treatments (i.e. qualified dividends, capital gains, etc.) retain their character when they flow-through to the owners rather than being reclassified as ordinary income or loss.

false

True or false: Interest income is generally taxed at lower capital gains rates.

savings bonds

U.S. do not pay periodic interest payments, but the interest accumulates over the term of the bond.

If taxpayer receiving life annuity lives longer than originally estimated

Ultimately receive more than expected number of payments. The entire amount of these "extra" payments is included in the taxpayer's gross income because the taxpayer has completely recovered her investment in the annuity by the time she receives them.

What is a qualified business?

Under the threshold amount, all non-employee trade or business income may be treated as coming from a qualified business.

Income from property is referred to as _______ income.

Unearned

When analyzing gains from property, explain Step 2 determining if gains (or losses) are recognized.

Unless otherwise provided, the gain or loss realized is recognized.

Gross income means all income from ____________ ____________ ______________.

Whatever source derived

Gross income means all income from _______ _________ __________

Whatever/Source/ Derived

recognize

When a taxpayer includes an economic benefit in gross income, he is said to have _____ the income.

Discharge of Indebtedness

When a taxpayer's debt is forgiven by a lender (the debt is discharged), the taxpayer must include the amount of debt relief in gross income

ordinary income

When donating _____ property as a charitable contribution, taxpayers can only deduct the lesser of (1) the property's fair market value or (2) the property's adjusted basis.

When is a discharge of indebtedness not included in gross income?

When the taxpayer is insolvent before and after the debt forgiveness

When analyzing gains from property, what is the analysis process and what must be determined?

You need to look at three different issues in determining if an amount is a long-term or short-term capital gain/loss. Must determine: 1) if you are dealing with a capital asset, 2) involved in a sell/exchange, 3) determine the holding period. Only assets held for "more than one year" receive the rate preference.

Irene's husband passed away this year. After his death, Irene received $250,000 of proceeds from life insurance on her husband, and she inherited her husband's stock portfolio worth $750,000. What amount must Irene include in her gross income? Zero but only if Irene does not opt to receive the life insurance proceeds in a lump sum. Zero - none of the above benefits is included in gross income

Zero - none of the above benefits is included in gross income

Shelly is a student who has received an academic scholarship to the University. The scholarship paid $4,000 for tuition, $500 for fees, and $400 for books. What amount must Shelly include in her gross income? $ 4,900 $ 4,000 $ 4,500 $ 4,400 Zero - none of the above benefits is included in gross income

Zero - none of the above benefits is included in gross income

Shelly is a student who has received an academic scholarship to the University. The scholarship paid $4,000 for tuition, $500 for fees, and $400 for books. What amount must Shelly include in her gross income?

Zero - none of the above benefits is included in gross income.

Shelly is a student who has received an academic scholarshipe to the Universality. The Scholarshipe paid 4K for tuition, 500 for fees, and 400 for books. What amount must shell belly include in her gross, disguisting, nasty, income?

Zeroh.

constructive receipt

_____ is deemed to occur when the income has been credited to the taxpayer's account or when the income is unconditionally available to the taxpayer, the taxpayer is aware of the availability and there are no restrictions on the income.

Which of the following statements is correct concerning a gift?

a gift may be subject to a "gift tax" which is paid by the person giving the gift

Court-ordered cash payments pursuant to a divorce or legal separation which provide financial support to an ex-spouse and do not continue after the death of the ex-spouse are referred to as

alimony

The term used when one former spouse is required to provide financial support to the other spouse pursuant to a legal separation or divorce is ______

alimony

items (from this chapter) included in gross income (7)

alimony and separate maintenance payments (property settlements, child support payments) annuity income group term life insurance imputed interest on below-market loans prizes and awards SS benefits unemployment compensation

front-loading of alimony payments

alimony recapture (gross income) for payor deduction from gross income for recipient

Beginning in 2019, in the case of a divorced couple, __________ payments are excluded from gross income of the recipient and ___________ for the payer.

alimony, nondeductible

Beginning in 2019, in the case of a divorced couple, _______ payments are excluded from gross income of the recipient and __________ for the payor

alimony; not deductible

income is earned when

all events have occurred that fix taxpayer's right to the income, and the amount can be determined with reasonable accuracy

flexible spending plans

allow employees to accept lower cash compensation in return for employer agreeing to pay certain costs without the employee recognizing income use or lose IRS allows a 2.5 month grace period to use funds for qualified expenses

qualified moving expense reimbursement

allowable moving expenses such as moving household goods/travel not amounts for meals, house-hunting, temporary housing allowances, and reimbursements for expenses related to sale of their prior residence may discriminate For AGI

cafeteria plans

allows employees to choose between cash and certain nontaxable benefits if cash is chosen, amount received is taxable

workers' comp

although may be payment for loss of wages, it is specifically excluded from gross income

acquisition indebtedness

amounts incurred to acquire, construct, or substantially improve the qualified residences interest paid on aggregate acquisition indebtedness of $1M or less (500K MFS) is deductible as qualified residence interest

Which of the following statements is correct concerning an inheritance?

an ineritance may be subject to the federal estate tax which is paid by the estate of the person who died

A _____________ is an investment that pays a stream of equal payments over time.

annuity

a(n) _______ is an investment that pays a stream of equal payments over time

annuity

life insurance investment earnings

arising from the reinvestment of life insurance proceeds are generally subject to income tax

working condition fringe benefits

benefit provided by an employer that would be deductible by the employee if the employee rather than the employer paid the expense (cell phone, work car) may discriminate

accident and health insurance benefits

benefits received under policy purchased by taxpayer are excludible different rules apply if accident and health insurance protection was purchased by individual's employer

Using scholarship, what can she exclude from gross income?

books, lab fees, tuition

biking commuting reimbursement

can exclude up to $20/month received from an employer as reimbursement for cost of commuting by bike (improvement, repair, storage)

3 primary methods of accounting for tax purposed

cash receipts and disbursements method accrual method hybrid method

The _______ _____ ______ doctrine states that income has been realized if a taxpayer receives income and there are no restrictions on the taxpayer's use of the income

claim of right

The ________________ doctrine states that income has been realized if a taxpayer receives income and there are no restrictions on the taxpayers use of the income.

claim of right

tax treatment of personal injury

compensatory damages received on account of physical personal injury or physical sickness are excludible (includes amounts received for loss of income associated with physical injury or sickness) all other personal injury damages are taxable (punitive, compensatory damages for nonphysical injury)

____________ is deemed to occur when the income has been credited to the taxpayer's account or when the income is unconditionally available to the taxpayer, the taxpayer is aware of the availability and there are no restrictions on the income.

constructive receipt

what to include

corporate bond interest, salary

Investments that are tax-advantaged when used to help fund higher education

coverdell savings account, section 529 plans, US Series EE bonds

interest income accrues _____

daily

alimony is deductible by the _____ and includible in gross income of ______

deductible by the payor includible by recipient *not for divorces after 12/31/18

Are ordinary losses deductible?

deductible with no limitation.

Rather than claiming the foreign-earned income exclusion, taxpayers may claim a foregin tax ______ or a foreign tax __________ for income taxes paid to other countries

deduction; credit

distance test

distance from old home to new job must at least be 50 miles farther than your old home was to your old job

What to include in income

dividend income, partnershipi income, judgment, salary

what dividends are not eligible for reduced tax rates

dividends from certain foreign corporations dividends from tax-exempt entities dividends that do not satisfy the holding period requirement

Income received as a result of services provided by the taxpayer, including business income, is referred to as _______________ income.

earned

Which ofo the following types of income is NOT generated from owning property

earnings from services rendered

Earnings on investments in plans such as Section 529 plans, Coverdell savings accounts, and US Series EE bonds are excluded from taxation if the proceeds are used for qualifying _______ expenses

educational

Nonrecognition provisions refer to a specific type of income that taxpayers realize but are allowed to permanently _________ from gross income of temporarily __________ until a later period.

exclude, defer

qualified moving expenses

excluded from AGI but no deduction for related expenses

exclusion formula for interest on education savings bonds

exclusion = (qualified educational expenses paid during year / total redemption proceeds of qualified bonds) x accrued interest

qualified parking

exclusion for 2017 is $255/month

SS benefits (limited extent)

exclusion from gross income

annuities (cost element)

exclusion from gross income

child support payments

exclusion from gross income

gain on sale of personal residence (with limitations)

exclusion from gross income

gifts received

exclusion from gross income

group-term life insurance (coverage up to $50,000)

exclusion from gross income

inheritances

exclusion from gross income

interest from state and local bonds

exclusion from gross income

life insurance paid on death

exclusion from gross income

meals/lodging (furnished for employer's convenience)

exclusion from gross income

municipal bond interest

exclusion from gross income

scholarship grants (limited extent)

exclusion from gross income

veterans' benefits

exclusion from gross income

exclusion ratio

exclusion ratio = (investment in contract) / (expected return under contract)

life insurance proceeds

exempt income to beneficiary if paid solely due to death of insured (relationship to decedent not determinative)

exceptions exist for policy transfers for life insurance

facilitate funding of buy-sell agreements pursuant to a tax-free exchange for receipt of a policy by gift

A tax payer who takes out a loan would put it in gross income TF

false

true or false: In general, prizes awarded to taxpayers are excluded from gross income

false

100% health insurance premiums paid by a self-employed individual

for AGI

contributions to pension, profit sharing, annuity plans, IRAs, etc.

for AGI

deductions for losses on sale of trade or business assets

for AGI

flow through entities

for AGI

interest on student loans

for AGI

penalty on premature withdrawals from time savings accounts/deposits

for AGI

qualified moving expenses

for AGI

qualified tuition and related expenses

for AGI

rent and royalty related

for AGI

trade or business expenses (self-employed)

for AGI

unreimbursed moving expenses

for AGI

up to $250 for teacher supplies for elementary and secondary teachers

for AGI

reimbursed employee business expenses

for AGI unreimbursed go into misc itemized (2%)

alimony paid

for AGI deduction

capital loss deduction (max $3,000)

for AGI deduction

certain payments to IRA and health savings account

for AGI deduction

interest on student loans

for AGI deduction

one-half of self-employment tax paid

for AGI deduction

qualified unreimbursed moving expenses

for AGI deduction

In addition to receiving a salary from a company, many employers provide ____ ______ that are excluded from gross income

fringe benefits

misc itemized deductions (excess of 2% AGI)

from AGI

personal casualty losses (in excess of 10% AGI and $100 floor per casualty)

from AGI

qualified mortgage interest

from AGI

qualified residence interest

from AGI

charitable contributions

from AGI deduction

interest on home motgages

from AGI deduction

investment interest (limited)

from AGI deduction

itemized deductions

from AGI deduction

medical expenses in excess of 7.5 of AGI

from AGI deduction

personal & dependency exemptions

from AGI deduction

personal property taxes

from AGI deduction

real estate taxes

from AGI deduction

standard deduction

from AGI deduction

state and local income or sales taxes (capped at $10,000)

from AGI deduction

if owner of life insurance policy cancels the policy and receives the cash surrender value

gain must be recognized to extent amount received exceeds premiums paid on policy loss is not recognized

accelerated death benefits

gain on cash surrender/transfer of policy by terminally or chronically ill individual is excludible (exclusion for chronically ill is limited to amounts used for long-term care)

tax treatment of damages received for loss of income

generally taxed the same as the income replaced exceptions exist related to personal injury

what does imputed interest apply to? (4)

gift loans compensation-related loans corporate-shareholder loans tax avoidance loans

When an individual transfers property to another taxpayer during their life, without receiving or expecting to receive value in return, the property transferred is a ____ and is not included in the ______ (income/property) by the person receiving the property

gift; nontaxable

_______ is defined in Sec.61 of the Internal Revenue Code as "all income, from whatever source derived".

gross income

Assignment of Income Doctrine

holds that a taxpayer who earns income from services must recognize the income as theirs

Tax Benefit Rule

if a refund is made for an expenditure deducted in a previous year, then the refund is included in gross income to the extent that a prior deduction produced a tax benefit

Which of the following statements is correct?

if a taxpayer receives a state tax refund for a tax year where she deductd the state tax paid, she must report the refund as gross income taxpayers who exchange or trade goods or services with each other must recognize the increase in value of the goods or services as income when taxpayers sell assets, they may exclude the original cost of those assets from gross income

life insurance policy transfers for valuable consideration

if policy is transferred for valuable consideration, proceeds are taxable to extent they exceed amount paid for policy + subsequent premiums paid

Taxpayers receiving indirect economic benefits, such as bargain purchases or below market loans, are said to have _____________ income which may be taxable.

imputed

imputed interest on below-market loans

imputed interest = difference between the amount that would have been charged at the federal rate and the amount actually charged

gift loans of $100,000 or less between individuals

imputed interest is limited to borrower's net investment income for the year no imputed interest if net investment is 1,000 or less

In general, when a taxpayer's debt is discharged by a lender, the taxpayer's gross income will ______ the forgiven amount

include

two alternatives if time test isn't met

include amount deducted in gross income in following year or file amended return for year of move or wait until time test is met and file amended return for year of move

amounts received for medical expenses deducted on a prior return must be ________ in gross income

included

payments for expenses that do not meet the code's definition of medical care must be ________ in gross income

included

alimony received

included in gross income

annuities income element

included in gross income

awards

included in gross income

bonuses

included in gross income

death benefits

included in gross income

gambling winnings

included in gross income

group-term life insurance, premium paid by employer (over $50,000)

included in gross income

hobby income

included in gross income

interest

included in gross income

non-qualified moving expenses

included in gross income

prizes

included in gross income

punitive damages

included in gross income

rents

included in gross income

unemployment benefits

included in gross income

unemployment compensation

included in gross income

payments that are a substitute for salary are _______ in gross income

includible

inheritances are nontaxable to beneficiary if

income earned on gifts or inheritances is taxable under normal rules

Imputed Income definition

income from an economic benefit taxpayer receives indirectly rather than directly. Amount of the income is based on comparable alternatives.

Claim of Right Doctrine

income has been realized if a taxpayer receives income and there are no restrictions on the taxpayers use of the income

taxability of income

income is recognized (taxed) when realized realized when (1) earnings process is complete (measurable change in property rights) and (2) an exchange or transaction has taken place

cash receipts method

income is recognized in the year it is actually or constructively received in cash or cash equivalent an amount is constructively received when it is set aside and made available to taxpayer without substantial restrictions

accrual method

income is recognized in the year that it is earned regardless of when it is collected the accrual method is required for determining purchases and sales when inventory is an income-producing factor

Which of the following statements is correct regarding the recognition of income?

income may be in the form of cash, property, or services received in a transaction

For below-market-loans, the discounted interest rate is treated as interest _______ to the lender and interest _________ to the borrower

income; expense

Form of Receipt Doctrine

indicates that taxpayers realize income whether they receive money, property, or services in a transaction

Income from Flow-through Entities

legal entities (partnerships, limited liability companies, and S corporations) that don't pay income tax. Income and losses are allocated to their owners. That is, owners report income or deductions as though they operated a portion of the business personally. Specifically, each partner or S corporation shareholder reports his or her share of the entity's income and deductions, generally in proportion to his or her ownership percentage, on his or her individual tax return.

compensation-related loan steps lender borrower

lender: 1) interest income 2) compensation expense borrower: 1) compensation income 2) interest expense

corporation to shareholder loan steps lender borrower

lender: 1) interest income 2) dividend paid borrower: 1) interest expense 2) dividend income

gift loan steps lender borrower

lender: 1) interest income 2) gift made borrower: 1) interest expense 2) gift received

what is the exemption amount for gift loans to be excluded from gross income

less than or equal to 10,000 this also applies to compensation-related and corporation-shareholder loans

transportation in commuter highway vehicle and transit passes

limit on exclusion for 2017 is $255/month

qualified tuition waivers or reductions by nonprofit educational institutions are excluded from income (2)

limited to undergrad tuition waivers exception for graduate teaching or research assistants

home equity indebtedness

loans secured by qualified residences interest is only deductible on portion of home equity loan that does not exceed the lesser of -100K (50K for MFS) or -FMV of home - acquisition indebtedness

subsidized eating facilities (de minimis) can be excluded if

located on or near employer's premises revenue equals or exceeds direct operating costs nondiscrimination requirements are met

If a taxpayer cashes out a life insurance policy before death due to a chronic illness, she may exclude from income the amount used to pay for her ________-_________ ________

long-term care

If a taxpayer cashes out a life insurance policy before death due to a chronic illness, she may exclude from income the amount used to pay for her_____________.

long-term care

taxpayer may seek damages for (4)

loss of income expenses incurred property destroyed personal injury

income from S corporations

may elect to be taxed similarly to a partnership the shareholders (rather than the corp.) pay the tax on corp.'s income

what does not qualify as moving expenses

meals, house hunting, temporary living, fees with purchase/sale of house

what qualifies as a moving expense

moving HH goods and personal effects to new location expenses of travel for taxpayer and family to new location -lodging -actual auto costs or mileage rate of .17/mile for each car

Interest in _________ bonds is excluded from federal income taxation.

municipal

Interest on ______ bonds is excluded from federal income taxation

municipal

interest on education savings bonds (4)

must be issued to individuals at least 24 years old prohibits gifts of qualified bonds must be used for qualified educational expenses (tuition, not books or room/board) of taxpayer, taxpayer's spouse, or dependents if redemption proceeds exceed education expenses paid in same year the amount of exclusion must be reduced

Are personal losses deductible?

never deductible.

qualified moving expense reimbursement new place of work mileage

new place of work must be 50 miles farther from former residence than was former place of work

child support payments are _____ by the payor and are ________ by the recipient

nondeductible by the payor not taxed by the recipient

In general, life insurance proceeds are _____ to the beneficiary of the policy

nontaxable

meals/lodging for convenience of employer

nontaxable employers may discriminate between employees

qualified employee discounts

nontaxable if: -discount is not on realty or investment property -item discounted is from same line of business in which employee workds -offered for sale in normal course of business -discount cannot exceed gross profit on property of 20% of the customer price on services -benefit is offered on nondiscriminatory basis

no additional cost services (4)

nontaxable if: -employee receives services -employer incurs no substantial additional cost in providing these services -services offered are within line of business in which employee works -benefit is offered on nondiscriminatory basis

scholarships/fellowships

nontaxable to extent of tuition and related expenses

tax treatment of reimbursement expenses incurred

not income, unless the expense was deducted damages that are a recovery of the taxpayer's previously deducted expenses are generally taxable under the tax benefit rule

amounts received from insurance are __________ when received for medical care or for permanent loss of body part or function

not taxable

premiums paid by employer for insurance coverage of employee, spouse, and dependents are ________ to employee

not taxable

stock dividends

not taxable if shareholder has the option to receive stock or cash, the dividend is taxable whether the shareholder receives cash or stock

Fixed annuities- expected value

number of payments times amount of the payment. In other words, for an annuity payable over a fixed term the return of capital is simply the original investment divided by the number of payments.

annuities with a fixed term - what is the expected value

number of payments x payment amount

Income from property

often referred as unearned income- income from property that accrues as time passes without effort on part of the owner of property. Gain or losses from the sale of property, dividends, interest, rents, royalties, and annuities.

How are corporate losses on capital gains restricted?

only the amount of gains

employee death benefits may be excludible as a gift if (5)

paid to surviving spouse or children employer derived no benefit from payments surviving spouse/children performed no services for employer decedent had been fully compensated for services rendered payments made pursuant to board of director's resolution under a general company policy

what payments qualify as alimony (4)

payments are in cash agreement or decree does not specify that the payments are not alimony payor/payee are not members of the same HH at the time the payments are being made there is no liability to make the payments for any period after death of recipient

trade/business deductions

permits deduction for all ordinary/necessary expenses paid or incurred in carrying on a trade/business salaries expenses for use of business property 1/2 self-emp. taxes paid all for AGI deductions

In personal injury cases, any damages awarded due to ______ injury are exempt from taxation

physical

Income from ___________ takes different forms, such as dividends, interest, rents, royalties, and annuities.

property

how is property treated in a marriage that was purchased during the marriage

property acquired by either spouse during the marriage is usually community property and is treated as though it is owned equally

form 1065 for partnerships (4)

provides data necessary for determining each partner's distributive share of partnership's income and deductions each partner reports distributive share of partnership income and deductions reported in year earned, even if not actually distributed because a partner pays tax on income as the partnership earns it, distributions are treated under the recover of capital rules

Which fringe benefits provided by an employer is excluded from gross income?

qualified transportation expenses, de minimis benefits, medical insurance

taxpayers report _____ and _____ income on their tax returns for the year

realized and recognized

Under the cash method, taxpayers recognize income inthe period the _______ it, rather than when they actually ____ i

receive; earn

When a taxpayer includes an economic benefit in gross income, he is said to have ______ the income

recognized

When a taxpayer includes an economic benefit in gross income, he is said to have _____________ the income.

recognized

reduction formula for interest on education savings bonds

reduction = (excess of AGI / $15,000 or $30,000) x otherwise excludable interest

tax benefit rule

refunds of expenditures deducted in a prior year are included in gross income to the extent that the refund reduced taxes in year of the deduction

income does not include recovery of the taxpayer's capital investment - 3 examples

repayment of loan sale or property at tax basis (adjusted basis) damages (however punitive and lost profits are taxable)

claim of right doctrine

requires amounts received to be included in income even though the amount is in dispute and might be returned to the payor at a later date if payment has not been received, no income is recognized until the claim is settled

how is property treated in a marriage that was purchased before the marriage

separate property by the spouse who purchased it

phase out amounts for single (HOH) and MFJ

single (HOH): $78,150 MFJ: $117,250

steps to compute annual taxable benefit

step 1: subtract 50,000 from death benefit of their employer-provided group-term policy step 2: divide step 1 by 1,000 step 3: multiple result from 2 by the cost per 1,000 of protection for one month from the table (based on age) step 4: multiply outcome in 3 by number of months the benefit was received during the taxable year

de minimis fringe benefits

supper money occasional personal use of company copying machine company cocktail parties picnic holiday gifts (ham, turkey) may discriminate

disability premium benefits are

taxable

scholarships for room and board taxable/nontaxable?

taxable

dividends

taxable to extent paid out of either current or accumulated earnings and profits dividends in excess of E&P are treated: -as nontaxable return of capital to extent of stock basis -as capital gain to extent in excess of basis

beneficiaries of estates and trusts

taxed on the income earned by the estates or trusts that is actually distributed or required to be distributed to them any income not taxed to the beneficiaries is taxable to the estate or trust

time test

taxpayer must be a full-time employee for 39 weeks in the 12 month period following the move OR self-employed must work in new location for 78 weeks during the next two years following the move (and 39 of those weeks must be in the first 12 months)

assignment of income doctrine (3)

taxpayer who earns income from services must recognize the income income from property such as dividends and interest is taxable to the person who actually owns the income-producing property to shift income from property to another person, a taxpayer must also transfer the ownership in the property to the other person

gain on sale of personal residence

taxpayers made exclude up to $250,000 ($500,000 for MFJ) of gain on the sale of their principal residence must satisfy ownership and use tests any excess gain generally qualifies as long-term capital gain

Constructive Receipt Doctrine

taxpayers realize and recognize income when it is actually received or was available to them

if contributor receives a tangible benefit from donating

the FMV of such benefit reduces the amount of chartable income contribution deduction

How do you compute depreciation for oil and gas property?

the deduction due to the exhaustion of natural resources is called depletion.

What are the tax consequences for a taxpayer who is receiving a life annuity and who has already lived longer than his or her life expectancy?

the entire amount of each additional payment of the annuity is taxable

Which of the following statements is INCORRECT regarding gambling winnings and losses for recreational gamblers?

the excess of gambling winnings over gambling losses are included in gross income

community property systems

the income earned from services by one spouse is treated as though it was earned equally by both spouses

Andres, Badin, and Carr formed a partnership, ABC. During year 2, the partnership sold some land that was held for investment and generated a long-term capital gain. How will this income be reported on the partners' individual tax returns?

the income will retain its character and be reported as a long-term capital gain

Which of the following statements is INCORRECT regarding workers' compensation payments?

the payments are included in gross income because they are a replacement of wages

Which of the following situations satisfies the ownership test for excluding the gain on a personal residence?

the taxpayer must have owned the residence for 2 or more years during the past 5-year period ending on the date of sale

What is the tax treatment for a taxpayer receiving a gold watch valued at #350 in recognition of his 25th year of working for the same company?

the value of the watch is excluded from gross income

how much of a deduction is allowed for athletic tickets? services performed?

tickets: 80% of amount paid services: nothing (can deduct unreimbursed and standard mileage of .14

two tests need to be met for moving expenses to be deductible

time and distance

education expenses of an employee are deductible if they are incurred

to maintain or improve existing skills or to meet express requirements of the employer or requirements imposed by law to retain employment status

education expenses are not deductible if they are incurred

to meet minimum educational standards for existing job or to qualify taxpayer for new trade or business

foreign earned income

to qualify for exclusion - must be either: a bona fide resident of foreign country OR present in foreign country at least 330 days during any 12 consecutive months

Under the realization principle, income is realized when (1) a taxpayer engages in a(n) _____ with another party, and (2) the ______ results in a(n) _______ change in property rights

transaction; transaction; measurable

gifts are nontaxable to recipient if (2)

transfer is voluntary without adequate consideration made out of affection, respect, admiration, charity, or donative intent

property settlements (4)

transfer of property to former spouse no deduction or recognized gain/loss for transferor no gross income and carryover of transferor's basis for transforee front-loading of alimony payments

tax treatment of property damaged or destroyed

treated as an amount received in a sale or exchange of property (taxpayer has realized gain if damage payments exceed property's basis)

true or false: Income and deductions generated within a partnership or S corporation that are subject to various tax treatments (qualified dividends, capital gains, ect.) retain their character when they flow-through to the owners rather than bing reclassified as ordinary income or loss

true

employee education expenses include

tuition, books, supplies, transportation, travel (including lodging and 50% meals)

Income from property is referred to as _______________ income.

unearned

Which of the following terms is used to refer to income from property?

unearned income

dependent care benefits

up to $5,000 exclusion for cost of providing care for a dependent under 13 years or disabled cannot be used for amounts paid to child of employee who is under 19 or any other dependent of the employee

education assistance programs (5)

up to $5,250 exclusion for tuition, books, and fees (not meals, lodging or transportation) excess amounts are taxed as compensation to employee amounts excluded from income cannot qualify for educational deductions or credits cannot discriminate for employees who are highly compensated does not cover educational payments for courses involving sports, games, hobbies

qualified retirement planning services

value of any retirement planning advice or information provided by employer who maintains a qualified retirement plan is excluded from income

athletic facilities

value of use of athletic facilities located on employer premises can be excluded

Gross income means all income from ________

whatever source derived

Return of Capital Doctrine

when receiving a payment for property, taxpayers are allowed to recover the cost of the property tax-free


Kaugnay na mga set ng pag-aaral

Solutions, Acids, and BasesWhich is an example of a solution?

View Set

Psicologia General 2 - 2do Parcial

View Set

training and development midterm

View Set

AP Classroom for Chemical Thermodynamics

View Set

The Four Magical Questions of Application 🦄

View Set